You are on page 1of 106

BUSINESS ORGANIZATIONS

COURSE OUTLINE

Prof. Robert Ragazzo Fall


2018

Contents
I. Agency................................................................................................................................................5
A. The Creation of an Agency Relationship.....................................................................................5

B. Four Types of Agent Authority....................................................................................................7

C. Principal’s Liability in Contract...................................................................................................7

D. Agents’ Liability to Third Parties..............................................................................................12

E. Third Parties’ Liability to the Principal......................................................................................12

F. Duties Agents Owe Principals (and Vice-Versa).......................................................................13

G. Imputing an Agent’s Knowledge to the Principal......................................................................14

H. Termination of Agency Relationship.........................................................................................15

I. Subagency..................................................................................................................................15

II. Partnerships.....................................................................................................................................17
A. Formation and Status.................................................................................................................17

B. Management of Partnerships......................................................................................................20

C. Sharing partnership profits:........................................................................................................20

D. Sharing partnership losses:........................................................................................................21

E. Apparent Authority in Partnership.............................................................................................21

F. Liability in Partnership..............................................................................................................22

G. Ownership Interests and Transferability in Partnerships............................................................23

H. Creditors of partners..................................................................................................................23

I. Duties Partners Owe Each Other................................................................................................24

J. Self-Dealing in the Partnership Context.....................................................................................25

K. Duty of Disclosure.....................................................................................................................25
L. More Duty of Loyalty................................................................................................................26

M. Leaving a Partnership................................................................................................................27

N. Dissolution of the Partnership....................................................................................................28

III. Corporations....................................................................................................................................30
A. Overview...................................................................................................................................30

J. How to Incorporate in Delaware................................................................................................31

K. Binding Corporations in Contracts.............................................................................................31

L. De Facto Corporation / Corporation by Estoppel (Delaware)....................................................32

M. De Facto Corporation / Corporation by Estoppel (Model Act)..................................................32

N. Ultra Vires Doctrine...................................................................................................................33

O. Management and Operation: Allocation of Power.....................................................................34

P. Quorum Rules:...........................................................................................................................35

Q. Authority in the Corporate Context............................................................................................35

R. Shareholder Voting....................................................................................................................36

S. How Shareholders Can Order Their Affairs Through Contract..................................................37

T. Amending Bylaws and Certificates............................................................................................39

U. Transferability of Shares............................................................................................................40

V. Statutory Close Corporations and Close Corporations...............................................................41

W. Piercing the Corporate Veil........................................................................................................42

X. Equitable Subordination.............................................................................................................45

Y. Duty of Officers and Directors...................................................................................................45

Z. The Business Judgment Rule.....................................................................................................46

AA. Duty of Care..............................................................................................................................47

BB. Self-Dealing...............................................................................................................................48

CC. Corporate Opportunity Doctrine................................................................................................50

DD. Executive Compensation (A Form of Self-Dealing)..................................................................51

EE. Majority Shareholders’ Duties...................................................................................................52

2
FF. Sale of Control...........................................................................................................................53

GG. Indemnification..........................................................................................................................55

HH. Duty that Shareholders Owe Each Other...................................................................................56

II. Shareholder Oppression: Dissolutions and Buy-Outs................................................................59

JJ. “No Special Rules” Jurisdictions (Contracts Trump Minority Shareholder Protection).............61

KK. Contract vs. Fiduciary Duty and Oppression.............................................................................63

LL. Dissolution for Deadlock...........................................................................................................63

MM. Securities Fraud.........................................................................................................................64

NN. Certificate Amendments............................................................................................................65

OO. Bylaw Amendments...................................................................................................................66

PP. Sale of Assets and Appraisal......................................................................................................66

QQ. Mergers......................................................................................................................................68

RR. Derivative Suits.........................................................................................................................70

IV. Limited Partnerships.......................................................................................................................75


A. Overview..........................................................................................................................................75

B. Defective Formation..................................................................................................................75

C. Entity Status...............................................................................................................................75

D. When is Limited Liability Lost?.............................................................................................76

E. Fiduciary Duties in Limited Partnerships..............................................................................77

F. Fiduciary duties when the general partner is a corporation: In Re USACAFES, L.P..................78

G. Duties of Limited Partners.........................................................................................................79

H. Ownership Interests and Transferability....................................................................................79

I. Exit Rights: Dissociation and Dissolution..................................................................................80

V. Limited Liability Companies..........................................................................................................83


A. Overview...................................................................................................................................83

B. Formation..................................................................................................................................83

C. De Facto LLCs...........................................................................................................................83

3
D. The Role of Contract..................................................................................................................84

E. Member-Managed versus Manager-managed LLCs..................................................................84

F. Authority in the LLC Context....................................................................................................85

J. Limited Liability: The Scope of Limited Liability.....................................................................86

K. Ownership Interests and Transferability....................................................................................89

L. Dissociation and Dissolution.....................................................................................................90

VI. Insider Trading................................................................................................................................92


A. Rule 10b-5.................................................................................................................................92

VII. Contests for Corporate Control......................................................................................................97


B. Proxy fights................................................................................................................................97

C. Tender Offers.............................................................................................................................97

4
I. Agency
A. The Creation of an Agency Relationship
a. R.3d of Agency § 1.01: An agent relationship is created when:
i. An agent is acting on behalf of the principal and subject to his control; and
ii. The agent and principal mutually agree that the agent will so act.
b. No contract or consideration necessary for an agency relationship to be created. The
parties don’t need to be consciously creating an agency relationship, either.
c. There’s no vicarious liability in an agency relationship.
i. Gorton v. Doty
1. A teacher lends her car to a coach for him to use to transport his student
athletes.
2. The Court finds an agency relationship and holds the teacher liable for
the coach’s negligence in driving the car.
a. Mistake 1: The coach was not acting for the benefit of the
teacher, so no agency relationship was created.
b. Mistake 2: It doesn’t matter whether the coach was an agent of
the teacher – there’s no vicarious liability unless there’s an
employer-employee relationship.
3. The teacher could have protected herself by:
a. Making the agreement in writing; and
b. Making it clear the car was a loan and didn’t mean she was a
principal.
i. The principal’s expressed intention isn’t dispositive, but
it would help.
d. However, a principal can become responsible for the agent’s debts.
i. R.2d Agency § 140: A creditor can also become a principal if it exercises control
over the debtor’s business.
1. There’s no requirement that the agent act on the principal’s behalf.
a. This is because money earned by the debtor will go to the
creditor until the debt is paid. In the case of Cargill, they own
$3.6 million in Warren’s debt.
ii. Jenson v. Cargill
1. Cargill (principal) lends money to Warren, and ends up exercising a lot
of control over Warren’s business.
a. They give “strong paternal guidance;”
b. They have access to all the agent’s information;
c. Principal had to give consent before agent could take out a
mortgage, guarantee debts of other parties, make big repairs, or
make distributions.
2. Eventually, as Warren continues to not make a profit to pay back the
loan, a guy from Cargill actually comes to the premises to control the
business.
3. Why does Cargill keep lending money to Warren?
a. The pot odds. Cargill is taking a 33% chance to win 10 times its
bet. The potential reward is sufficient to justify the risk

5
4. The shift from creditor to principal occurred when Cargill evolved from
having veto power to affirmative control.
5. Having found that Warren is Cargill’s agent, the court finds that
Warren’s debts are Cargill’s – including both Cargill’s loan and loans
from third parties.
6. How could Cargill have prevented this disaster?
a. Only exercise negative control, not affirmative control.
e. Policy: Is the rule enunciated in Cargill good or bad policy?
i. It increases every company’s cost of borrowing money (because it increases risk
for lenders, thus requiring higher interest rates).
ii. However, it prevents lenders from forcing borrowers to make risky investments
to pay the creditors back.
1. E.g. – Tony Soprano takes you to a casino and tells you to win enough
money to pay him back or he’ll break your legs. You will choose to play
roulette rather than blackjack because there’s a higher potential reward
from roulette, although the odds of winning are much lower.
2. It’s society’s way of telling the Cargills of the world that they have to
take on the debts accrued by debtors who are acting on the creditors’
orders in order to pay back the creditors.
a. Cargill can’t play roulette with Warren’s money with no risk to
itself.
f. R.2d Agency § 14K: A supplier becomes an agent if the supplier agrees to act on the
customer’s behalf rather than his own.
i. Two of three factors in Comment A of § 14K suggest that Warren is a supplier,
not an agent.
g. Agency relationships are task-specific.
i. Green v. H&R Block
1. Facts:
a. H&R Block prepares taxes, but also offers “Rapid Anticipation
Loans” (RALs) that give a discounted refund in the form of a
loan from a bank. The bank is paid back with the taxpayer’s
refund.
b. H&R Block didn’t disclose that they’re getting a kickback from
the banks and Sears, and that they’re buying some of the loans
themselves.
2. Issue: Did H&R breach a duty to the customers by failing to disclose
their conflict of interest in setting up the RALs?
a. If H&R were the customers’ agent in setting up the loans, then
there’d be a fiduciary relationship, and H&R would have to
disclose these conflicts of interest.
b. It’s not a sure thing, however, that H&R is an agent even in the
tax preparer context.
i. Even if it were an agent in that task, their duty ends with
the tax preparation and doesn’t extend to the RALs.
c. When setting up the RALs, H&R is an agent of the banks.
They’re screening customers for them.

6
3. Rule: You can be a “dual-agent” if all you’re doing for one of your
principals is a “ministerial task.”
a. In this case, giving the taxpayer the paperwork to sign up for an
RAL is ministerial.

B. Four Types of Agent Authority


a. Express Actual Authority
i. A principal has expressly communicated to an agent the power to perform some
act on the principal’s behalf.
b. Implied Actual Authority
i. An agent has implied actual authority when the principal does not expressly
confer authority but the principal’s words or conduct, reasonably interpreted,
causes the agent to believe that he has authority.
ii. TEST: Would a reasonable person in the agent’s position reasonably believe he
had the authority to do a thing on the principal’s behalf based on the principal’s
manifestations?
c. Apparent Authority
i. An agent has apparent authority in dealing with a third person when the
principal’s words or conduct reasonably interpreted, causes the third person to
believe that the agent has authority. Can also be derived from prior practice or
industry custom.
ii. TEST: Manifestations by the principal that would lead a third party to reasonably
believe that the agent has authority.
1. Manifestations must come from the PRINCIPAL and NOT the AGENT;
therefore, this authority cannot exist if the principal is undisclosed!
iii. Often overlaps with implied authority, but it depends on whether the third party
knows what the agent knows.
d. Inherent Authority
i. A principal may be liable in contract for the acts of a general agent even if the
agent lacks actual and apparent authority.
ii. Exists in R.2A, but not R.3A.
iii. TEST: Is the power being exercised incidental to express authority that is not
specifically disallowed? [CHECK ON THIS]
iv. Disclosure of Principal:
1. Disclosed Principal: A principal is disclosed if the third party is aware
that a principal exists and knows the principal’s identity
2. Partially Disclosed Principal: A principal is partially disclosed if the third
party knows that a principal exists but is unaware of the principal’s
identity
3. Undisclosed Principal: A principal is undisclosed if the third party is
unaware that a principal exists and believes that he is doing business with
the agent
v. General Agent: A general agent is an agent authorized to conduct a series of
transactions involving a continuity of service
vi. Inherent Authority: A general agent has inherent authority to act for a disclosed
or partially disclosed principal regarding acts done on the principal’s account

7
which usually accompany or are incidental to transactions which the agent is
authorized to conduct if, although they are forbidden by the principal, the other
party reasonably believes that the agent is authorized to do them and has no
notice that he is not so authorized.

C. Principal’s Liability in Contract


a. If the agent has any kind of authority, the principal can be bound by a contract made by
the agent.
b. Express Actual Authority must be granted with specificity:
i. Williams v. Dugan
1. Issue: Did Edward have authority to borrow money to pay the principal’s
taxes?
a. The court says no because the blanket grant of authority to make
payments concerning the principal’s real estate investment did
not specifically allow borrowing money.
i. Courts don’t like enforcing blanket grants of authority.
ii. Ragazzo says this is a dubious contention because the
agreement allowed Edward to mortgage the property,
which is essentially borrowing money.
b. There was implied authority anyway because he was allowed to
take care of financial matters related to the property.
i. This doesn’t sway the court, however, because they want
a specific grant of authority.
2. Rule: Express grants of authority are strictly and narrowly construed.
a. Downside of this rule: You have to think of every specific
contingency for which the agent may need authority while
drafting an agreement.
ii. Implied/Apparent Authority
1. Essco Geometric v. Harvard Industries
a. The contract at issue was a two-year exclusive supply agreement
with Diversified Foam Company initiated by Gray on Harvard’s
behalf.
b. Issue: Did Gray have authority to bind Harvard to the contract?
c. Did Gray have express authority?
i. No. Two memos from the president say that any contract
must be approved by him first.
d. Did Gray have implied authority?
i. The court says this is a fact question. Ragazzo can’t
understand why the court would say this – no reasonable
person would think he had authority to enter a contract
after two memos told him he couldn’t.
e. Did Gray have apparent authority?
i. “No” Factors:
1. Non-cancellable, exclusive contracts are not the
norm in this industry.
ii. “Yes” Factors:

8
1. There had never previously been a requirement
for the president’s approval.
2. Purchasing agents are usually allowed to issue
contracts on their own.
3. While exclusive contracts aren’t the norm, in
practice, one supplier will typically get the lion’s
share of the business.
a. Furthermore, the principal had almost
never cancelled a contract before.
iii. All told, the court is right to say apparent authority is a
jury question in this case – it’s a deviation from prior
practice, but not a big one.
f. What should the president have done?
i. Tell the third parties about the new policies, so there
would be no apparent authority.
g. What should the supplier have done?
i. Always get corporate officers to tell you in writing what
they’re authorized to do. This will ensure they at least
have apparent authority to enter into contracts with you
on their principal’s behalf.
1. Go to the secretary and ask him to certify that
someone has authority to do something.
2. Whatever else is true, a secretary at least has
apparent authority to certify corporate records.
ii. HOWEVER, the supplier would have lost this case had
they done that because they would have found no
apparent authority. Sometimes ambiguity is better.
iii. Inherent Authority
1. Kidd v. Thomas A. Edison, Inc.
a. A singer was promised a deal to go on a singing tour.
b. Maxwell said Fuller was allowed to book her tours and get the
executives to agree to pay, not to actually contract with the
singers for singing tours.
c. R.2A § 3: General agents are employed for a series of
transactions in continuity of service, while special agents are not
engaged in a continuity of service.
i. Only general agents have inherent authority.
d. In this case, the test for inherent authority overlaps entirely with
the test for apparent authority.
e. In the R.3A, there’s no inherent authority when the principal is
disclosed.
i. It remains intact for situations with an undisclosed or
unidentified principal.
2. Morris Oil Co. v. Rainbow Oilfield
a. This case involves a contract for diesel fuel and a bulk dispenser.
b. There was an agency relationship between Rainbow (agent) and
Dawn (principal).

9
i. The substance of the deal was that all gross receipts go
to Dawn, who takes out their fees, then sends the
remainder to Rainbow.
ii. Rainbow operates under Dawn’s license.
c. What was the real relationship?
i. If Dawn keeps 5%, it seems like Rainbow is in business
for itself.
ii. If Dawn keeps 90%, then Rainbow is running the
business for Dawn.
iii. The court doesn’t say what percentage Dawn keeps, so
we don’t know which scenario is true.
d. Express Authority?
i. No. Rainbow is authorized to incur debt or liability for
the ordinary course of business relative to terminal
management.
ii. Arguably, the bulk loader purchase was not part of the
ordinary course of business – it’s a capital expenditure.
e. Implied Authority?
i. No. Rainbow didn’t have implied authority to buy fuel
or the bulk loader, either, because the contract
enumerated what they’re allowed to do and says nothing
else is authorized.
f. Apparent Authority?
i. No. The principal is undisclosed. There can be no
apparent authority when the principal is undisclosed (no
way for a third party to know an agency relationship is at
play).
g. Inherent Authority?
i. Maybe. It’s customary in the industry to buy fuel and
make capital improvements such as the bulk loader.
ii. Rainbow is acting as though it has Dawn’s resources.
Therefore, a third party would reasonably assume that
Rainbow had the size and resources to enter into big
contracts.
h. Morris might be able to collect on the contract in any case
because of the doctrine of estoppel.
c. Ratification:
i. The principal must have been able to give authority at the time of the original
contract for the principal to be able to ratify it.
1. The ratification makes it as though the contract were assented to at the
outset – the contract is entered into at the original date, not the date of
ratification.
ii. Evans v. Ruth
1. Facts:
a. The foreman of a stone quarry hired someone to haul stone,
unaware that a subcontract already existed for that job.

10
b. The only way the foreman would have been able to know about
the subcontract was if the principal (Mr. Ruth) had told him.
2. The foreman probably had implied actual authority to make contracts for
stone hauling.
3. He probably also had apparent authority because a reasonable third party
would reasonably believe the foreman had authority to give contracts.
4. Issue: Was there ratification?
a. Express ratification?
i. Mr. Ruth assented to the contract when the contractor
came and talked to him, therefore he expressly ratified it.
b. Implied ratification?
i. Yes, because the circumstances (esp. the receipts from
the foreman’s contract) indicate Mr. Ruth knew about
the contract and tacitly accepted it.
a. Botticello v. Stefanovicz
i. Facts:
1. The Stefanoviczes were joint tenants of a parcel of land. Walter S.
sold all of the land to Botticello.
2. Botticello thought Walter S. owned the whole property and B was
buying all of it.
3. Walter did not have authority to sell Mary’s half of the property.
ii. Express?
1. No. Mary said she wouldn’t sell for less than $85k – this is not an
express instruction to sell.
iii. Implied?
1. No. Reasonable person wouldn’t have thought he had authority.
iv. Apparent?
1. No. The principal is undisclosed. There can be no apparent authority
in the case of an undisclosed or unidentified principal.
v. Inherent?
1. No. Walter is not Mary’s general agent.
vi. Is there ratification? What kind?
1. Implied?
a. No. Botticello had a lease with an option to purchase. Mary
didn’t know about the option to purchase.
2. However, what about the improvements made by Botticello? It’s not
unthinkable that a long-term tenant would make improvements,
though there was no prospect of owning the property.
a. Implied ratification always requires knowledge on the part of
the principal.\
b. Hoddeson v. Koos Bros.
i. The π was trying to buy furniture, but the salesman was an impostor and stole
her money.
ii. Did the impostor have apparent authority?
1. No, because the reasonable belief must come from a manifestation of
the principal.
iii. What about estoppel?

11
1. To win on estoppel, the π must prove that the principal carelessly
caused the π’s belief.
iv. How to prove that the principal was careless?
1. Prove that an employee noticed something suspicious and didn’t do
anything. All of the employees’ knowledge is imputed to the
employer.
v. Why does the π sue under contract rather than tort?
1. The remedy for a tort claim – as opposed to a contract claim – is
getting the money back, nothing else (i.e. – no specific performance
or anything like that).

D. Agents’ Liability to Third Parties


a. If an agent isn’t careful, he can find himself bound under a contract he sets up on his
principal’s behalf.
i. Under R.2A, when an agent signs a contract on behalf of a partially disclosed
principal, liability is joint and several.
1. When the principal is disclosed, there’s alternate liability.
ii. Under R.3A, liability is joint and several unless the principal is fully disclosed.
iii. Van D. Costas, Inc. v. Rosenberg
1. Facts:
a. An agent sets up a contract on behalf of a restaurant chain with a
third party.
b. The agent finds himself bound under the contract because he got
the name wrong when identifying his principal.
2. Why does the third party care who’s bound under the contract?
a. They want to know the credit worthiness of the other party.
3. Why is Rosenberg (agent) bound under the contract?
a. Because he didn’t disclose the principal. He only gave the trade
name of the restaurant, rather than the corporate name of the
entity that owns it.
i. Why doesn’t the trade name count?
1. The third party needs the correct legal name of
the entity to check its credit-worthiness.
4. The restaurant is also bound because Rosenberg had express actual
authority or implied actual authority from the restaurant.

E. Third Parties’ Liability to the Principal


a. A third party can sue an agent if the agent and principal knew the third party would not
have entered the transaction had he known who the principal was.
i. Hirsch v. Silberstein
1. Facts:
a. Silberstein (agent) buys property from Hirsch, then transfers it to
a black family (principal).

12
b. Hirsch sues, saying he thought he was dealing only with the
Silbersteins, and that he never would have sold the property to
the principal.
c. Hirsch insisted this was not because they were black (so as not to
run afoul of fair housing laws).
2. Hirsch is estopped from arguing that the principal and agent knew about
his biases because he stipulated that he had no racial prejudices.
3. The result is a disaster for Hirsch because he is going to be neighbors
with the black family.
a. How to protect Hirsch from this result?
i. Put in the contract that the signing party is the principal.
4. The contract did have a no-assignment clause, but the Silbersteins got
around it because they sold the land to the black family rather than
assigning the contract.
a. Alternately, and even better, the Silbersteins are the agents of the
black family, so the black family is already a party to the
contract.
i. The Silbersteins had authority to sign for them.

F. Duties Agents Owe Principals (and Vice-Versa)


a. Agents’ Duties
i. Duty of Loyalty
1. Must put the principal’s interests above the agent’s.
ii. Duty of Care
iii. Duty of Obedience
1. Must be subject to principal’s control.
b. Tarnowski v. Resop
i. Facts:
1. The agent is supposed to investigate some routes for the principal to
purchase.
2. However, he did a superficial job and was lied to by the seller.
3. It was a complete fabrication that the agent didn’t discover.
4. Agent sued seller and got back the principal’s money.
5. Principal sued agent anyway.
a. Agent got a secret commission from the seller. That is the
Principal’s money.
ii. Rule: Duty of loyalty requires that, beyond the agent’s compensation, every
benefit flowing from the transaction must go to the principal.
1. Policy: The duty of loyalty has a preventive effect in that it takes away
the agent’s incentive to double cross the principal.
2. Even if the agent had done his job perfectly, he would still owe the secret
commission to the principal.
3. If the seller had been judgment proof and the principal couldn’t be made
whole from suing him, the principal can sue the agent for the
commission AND for the damages from the fraud.

13
iii. If there hadn’t been a kickback, the principal could still sue for breach of the duty
of care.
c. Principal’s Duties
i. Duty of Compensation/Indemnification
1. The principal must pay the agent for his services.
a. This is the other side of the idea that the agent must turn over all
benefits of the transaction to the principal – that’s what the
money’s for.
b. All principals have a duty to indemnify agents for all costs of
things done on their behalf.
c. Things agents should be compensated for:
i. Reasonable expenditures related to the agent’s duties.
d. Things agents shouldn’t expect compensation for:
i. Mailing a letter for a friend.
ii. Borrowing a car to take to a football game.
iii. Basically: Things that are personal in nature, not related
to agent’s duties.
2. McCollum v. Clothier
a. Facts:
i. The principal’s attorneys told the agent to “keep up the
good work and line up buyers” for the sheriff’s sale.
ii. The agent did a good job and got a good price for the
property at the sheriff’s sale.
iii. The principal is upset, and says that the agent didn’t do
what he was told to do.
1. The principal was actually upset because he
didn’t tell the agent to go on plane trips and such
– it went beyond what the principal was
expecting.
b. Rule: Principal’s have a duty to compensate agents and
remunerate them for all reasonable expenditures.
c. The agent didn’t make extravagant plane trips, but the principal
thought it was unreasonable.
i. It wasn’t unreasonable, so the principal must pay.

G. Imputing an Agent’s Knowledge to the Principal


a. Rule: An agent’s knowledge is imputed to the principal for most purposes.
i. Exception: When the agent is acting adversely to the principal’s interest.
ii. Kirschner v. KPMG LLP
1. Facts:
a. The corporation is suing its accounting firm for not catching the
fraud committed by the corporation’s own officers.
b. The corporation’s theory is that, but for the accountants’ failure
to catch the corporation’s own fraud, the corporation would not
have gone bankrupt.

14
c. The corporation is in the wrong because it has committed
massive fraud in its financial statements.
2. The accountants defend on In Pari Delicto grounds: A guilty plaintiff
can’t recover.
a. The success of the In Pari Delicto defense depends on whether
the officers’ knowledge of the fraud is imputed to the
corporation.
3. The Court finds that the agent must be acting WHOLLY for his own
benefit for the imputation exception to apply.
a. The corporation benefitted from the agents’ fraud in numerous
ways (healthier-looking balance sheets, improved stock price,
etc.)
b. The agents benefitted from the improved stock price and from
the success of the company improving their own marketability as
executives.
4. In a mixed-motivation case of fraud such as this, the agents’ knowledge
of the fraud is imputed to the principal.
a. The extent to which agents’ knowledge is imputed to the
principal is very important because a corporation knows nothing
except for what’s imputed from their agents.
5. The Corporation is clearly guiltier in this case, so there’s no argument
the accounting firm is more at fault because they negligently failed to
catch the fraud.
6. The Court found that it’s a more compelling public policy goal to
discourage companies from hiring bad apples who commit fraud than
making sure accountants do their jobs well.

H. Termination of Agency Relationship


a. There can be a time limitation or a transaction-based limitation on the duration of an
agency relationship.
b. There are also implied things that cause terminations.
i. R.2A lists a bunch of them.
ii. R.3A just says it terminates when an agent would reasonably understand that
changed circumstances in the world terminate his authority.
c. An agency relationship is also terminable at will by either agent or principal at any time,
even if it’s a breach of contract.
i. That doesn’t mean the breach is free – you’re liable under the contract for the
breach. However, the agency relationship can’t be forced – no specific
performance.
d. Agency Coupled with an Interest
i. Lane Mortg. Co. v. Crenshaw
1. I-Bankers fund a hotel company’s purchase of land. In exchange, they
get a free 20-year lease of the hotel’s second floor.
a. The I-bankers are also managing the property and collecting rent.
2. Rule: When an agency relationship is coupled with an interest, the
relationship is not terminable at will.

15
3. This isn’t really an agency power because the action (foreclosing on the
bank’s security interest) isn’t taken for the principal’s benefit.
a. It takes the form of an agency power, though.
4. If the agent or principal dies, the agency relationship is terminated, and
the creditor is just a regular creditor.
a. However, if you take a security interest in the property, it does
not depend on agency and the interest would survive termination
of agency.

I. Subagency
a. Subagents owe the normal agent’s duties to their principal AND all of the principal’s
principals.
b. Select Creations, Inc. v. Paliafito
i. Facts:
1. MAI entered an agency agreement with Paliafito, who then acquired a
subagent (Select Creations), who then acquired Forman as a subagent,
who joined with Composto as a coagent.
a. MAI -> Paliafito -> Select -> Forman -> Composto
2. Forman and Composto decided to cut out the middle man and go directly
to MAI for the work.
3. Paliafito sued Forman and Composto for breach of the fiduciary duty of
loyalty.
ii. Rule: Every subagent down the line owes a duty to every principal above them.
1. There’s no doubt that Forman and Composto are subagents of Paliafito
and, therefore, they owe a fiduciary duty to Paliafito.
2. Forman must remain loyal to each principal. If its immediate principal
told him to act disloyally to that principal’s principal, then Forman
doesn’t have to obey.
iii. Paliafito does not owe a duty of compensation to Forman – that’s Select’s duty.
1. Paliafito doesn’t know Forman exists, so he can’t be liable to Forman.

16
II. Partnerships
A. Formation and Status
a. It’s the only form of business organization that involves two or more people and doesn’t
involve filing papers with a secretary of state.
b. You don’t have to have anything in writing, although it is a good idea.
c. Features of a partnership:
i. All partners share equally in the profits and control of the partnership unless
otherwise stipulated in a contract.
ii. Partners are fully liable for the partnership’s liabilities – this can’t be changed by
contract.
d. Martin v. Peyton
i. Facts:
1. A business venture’s lenders are sued by other creditors for the debts of
the venture on the theory that the lenders are actually in partnership with
the business venturers.
2. The creditors want the lenders to be found to be partners because that
would make them liable for the firm’s debts.
ii. Court applies four factors:
1. Are losses shared?
2. Are profits shared?
3. Is control shared?
4. Community of interest? (Ragazzo says this is meaningless – covered by
the sharing of losses and profits).
iii. The above isn’t an exclusive list. If one is missing, it’s not dispositive.
iv. Did the lenders agree to share profits?
1. They gave liquid securities to the firm so that the firm could borrow
money.
2. There was a $100,000 floor and a $500,000 ceiling between which the
lenders will get 40% of net profits.
3. The floor and ceiling cut in favor of it not being a partnership.
4. The 40% cuts in favor of there being a partnership.
v. Did the lenders agree to share control?
1. They had extensive veto power – the could veto the firm’s business
decisions.
2. They also had the right to see the firm’s books and records.
3. They required the current management to remain in place.
4. These cut against partnership because negative power and information
power are not enough to establish shared control.
vi. Court remands for further fact-finding. It’d be a disaster for the lenders to be
found partners – they’d lose their own claim for repayment and have to repay
other creditors.
vii. How to make sure the lenders aren’t partners?
1. Share less of the profits. Not a percentage – ask for a hard figure or a
percentage of the loaned money.

17
2. It’s a business decision whether you want to take the substantial risk that
someone will think you’re in a partnership.
e. Partners owe one another a fiduciary duty to be fair in their dealings with each other.
i. It’s not a true fiduciary duty, because it isn’t a duty to be selfless and put one’s
partners’ interests above one’s own.
ii. Partners owe a duty not to steal each other’s business and a duty to inform the
partners of business opportunities.
f. Beckman v. Farmer
i. Facts:
1. Farmer wants to be deemed a partner so that he’ll get part of a multi-
million-dollar contingency fee.
2. Drafts of formal partnership agreements were exchanged, but there was
no finalized contract.
3. There was an agreement that Farmer would get a guaranteed “draw” of
$85,000 per year, plus a certain percentage of firm profits if certain
levels of net profit for the firm were met.
4. The firm was called “Beckman v. Farmer.”
ii. Did Farmer share profits?
1. Conditionally. You could make an argument he was an employee with a
bonus profit-sharing incentive on the side.
iii. Did he share losses?
1. His name is on the lease and such.
2. To date, though, Beckman paid for every liability.
3. Also, they aren’t sharing losses generally.
4. Formally, Farmer is sharing losses but, as a practical matter, he is not.
iv. Did he share control?
1. Beckman wanted to form a corporation to hold the lease because, if
membership changed, the partnership would be dissolved. However, he
allowed others to veto the decision to form a corporation.
2. Beckman also fired people, cut people’s hours, etc. without consulting
with anyone first.
3. Beckman said he was just consulting the “employees” to be collegial.
v. Other indicia:
1. They filed their taxes as a partnership.
2. Farmer’s name is on the door (“Beckman and Farmer”).
a. The way they’ve held Farmer out to the world as a partner counts
for a lot.
vi. Without question, Farmer is at least a partner by estoppel.
vii. Had the firm not held out Farmer as a partner, then Beckman would have a lot
easier time saying Farmer wasn’t a partner.
1. Also, Beckman should have had Farmer sign something saying he’s not a
partner.
g. Partnership by Estoppel
i. Cheesecake Factory, Inc. v. Baines
1. Facts:
a. Baines worked with Cheesecake Factory on behalf of a business
venture seeking to establish a sports bar.

18
b. Mr. Kolk, the sportsbar owner, represented to CF that Baines
was his partner.
i. Kolk probably didn’t understand the legal implications.
2. Cheesecake Factory can get to Baines to collect on the sports bar’s debts
if Baines is a partner.
3. CF argues that the New Mexico statute on partnership by estoppel
requires reliance for the cases of private representations, but not in the
case of public representations.
a. The Court says the purpose of the “public representations” part
of the statute is just so that the representation needn’t have been
made specifically to a particular creditor. The reliance
requirement remains.
4. At common law, reliance is required for both public and private
representations. The Revised UPA eliminates this ambiguity.
5. This has to be a clear-cut case of public representation. If Baines publicly
holds himself up as a partner, it doesn’t matter if there was implicit
authorization from Mr. Kolk.
6. Factors cutting against reliance:
a. CF did not run a credit check on Baines.
7. Factors in favor of reliance:
a. CF wanted Baines to have skin in the game because a partner
will probably work harder in running the business than a
shareholder.
i. The bar for what constitutes reliance is very low.
h. A partnership is dissolvable at will by any partner.
i. Under the UPA, any scenario where a partner walks away from the partnership
results in its dissolution (see UPA § 29).
1. The UPA embraces an aggregate theory of partnership.
2. The policy logic is that new partners can’t be liable for debts that existed
before they got there.
3. This is a regulatory rule – the UPA forbids an agreement that a
partnership will survive a change in membership.
4. However, if the outgoing and incoming partners all agree to maintain the
partnership, then it won’t be dissolved.
ii. The RUPA changes this, so that the default rule is that a partnership survives a
change in membership.
1. RUPA shifts to an entity theory of partnership.
iii. Fairway Development Co. v. Title Insurance Co.
1. The development company sued their insurance provider for failure to
honor their insurance agreement.
2. The insurance company argued that the partnership that signed the
original insurance agreement was dissolved, so there’s no one to pay.
a. If it’s a new partnership suing the insurance company, then they
are not a party to the insurance policy and have no standing to
sue.
3. The old partnership dissolved when Sara and Wenger ceased to be
partners. So, the insurance company is right, and the new partnership has

19
no standing to sue. They don’t have to pay out under the policy because
there’s no one to enforce the old partnership’s rights under it.
4. The lawyer for Fairway should have drafted the contract such that the
beneficiary is the old partnership and its successors in interest.
5.

B. Management of Partnerships
a. Bringing on additional partners must be approved by the partnership.
i. In Texas, voting power is proportionate to the partners’ share of the profits.
b. Summers v. Dooley
i. Dooley is mad at Summers for hiring a third partner.
1. Summers wanted to expand the business, but Dooley didn’t think it was
justified.
ii. All principals have a duty to indemnify agents for all costs of things done on
their behalf.
iii. The partnership owes the third partner 100% of his salary (remember that it’s the
partnership that pays, not the partners).
iv. Summers did not have express actual authority to hire the third partner.
1. There had to be a majority vote to bring on a new partner.
v. Summers also lacked implied actual authority.
1. A reasonable person wouldn’t have thought he was allowed to bring on
additional partners unilaterally.
vi. Summers’ argument: Summers was the managing partner, so he had delegated
authority to hire partners.
1. However, Summers asked Dooley if he could bring on the partner – this
shows Summers knew he lacked authority.
vii. Did Summers have apparent authority?
1. Standard: Is the action in the context of ordinary partnership business?
2. In this case, it’s not out of the ordinary to bring on a third partner to
handle partnership business.
viii. Dooley is liable for everything Summers did as to the outside world (i.e., the
partnership must pay the third partner).
1. Internally, Summers is liable to Dooley for illegitimately hiring a third
partner.

C. Sharing partnership profits:


a. Schymanski v. Conventz
i. The partners want to dissolve the partnership.
1. The partnership must first pay back creditors, then distribute the capital
account to the partners.
2. A capital account is the money contributed to the business, plus
reinvested profits, minus amounts distributed to the partners.
ii. The default rule for distributing profits is “one partner = one share of the profits.”
iii. The issue in this case is whether Conentz’s labor counts toward his contribution
to the business.

20
1. If so: 350 in revenue MINUS 244 in capital account = 106 profit equally
distributed to Schymanski and Conventy.
a. This is so because S = 134+53 = 187
b. C = 40+70+53=163
c. C+S=350
2. If not: 350 in revenue MINUS 174 in capital account = 176 shared
equally between C and S.
a. S=134 + 88 = 222
b. C=40+88=128
3. If the labor is counted, C gets more money during the winding up of the
partnership.
iv. The trial court found that C’s labor counted, so S was awarded the difference
between their contributions (approx. $24,000), and the profits were split evenly.
v. Default rule: Services aren’t compensated or counted toward the capital account
unless there’s an express or implied agreement otherwise.
1. In this case, the original agreement said the partners would pay equal
amounts into the business. A subsequent agreement said Conventz would
work on construction.
2. Conventz put in way less money than S, yet S didn’t complain, implying
S accepted the services as C’s capital contribution.

D. Sharing partnership losses:


a. Kessler v. Antinora
i. Kessler and Antinora made a partnership for the purpose of building a house.
1. K put in 500k.
ii. The default rule on losses is to share them in the same proportion as profits (i.e. –
one partner = one share)
iii. K says the losses should be split 60-40 in K’s favor.
1. An argument could be made that K should bear all the losses – he was
the money man while A is the service partner.
2. A is also not paid for his services, so he has also taken a loss in the form
of unpaid labor.
a. A wouldn’t even break even until the partnership had made
$750k in profit – the profit side is clearly tilted toward K.
Therefore, the loss side should be tilted toward A.
iv. These cases are typical of a trend where courts, when a business is profitable, the
service partner gets capital credit and where the business has losses, the service
partner isn’t liable.
1. One might say that the default rule is practically overturned by this trend.

E. Apparent Authority in Partnership


a. Burns v. Gonzalez
i. Mr. Bosquez signed a promissory note on behalf of a radio partnership. The
partnership does not want to be bound by this promissory note.
ii. The court tweaks the apparent authority rule for use in the partnership context:

21
1. Test: Is the action in the ordinary course of a partnership’s business in
this kind of business in a similar location?
iii. If you borrow money all the time to conduct normal business operations, then
borrowing money is in the ordinary course of business and there is apparent
authority to borrow money.
iv. On the other hand, if you only borrow money when you’re in trouble, then it’s
not in the ordinary course of business and there’s no apparent authority.
v. Partnerships that sell radio time don’t have cyclical periods where they need to
borrow money in anticipation of making profits later (like a farm) – they are a fee
for service business.
1. Therefore, borrowing money is not in the ordinary course of business and
Bosquez did not have apparent authority to sign the promissory note.
vi. Because there’s no apparent authority, the partnership isn’t bound by the
promissory note. (A subsequent agreement extinguished the π’s claim, so this is
all dicta).
b. Sheridan v. Desmond
i. Desmond’s agent repeatedly blocked his tenants’ fire exits, which was an attempt
to constructively evict them by getting gthem shut down by the fire marshall.
ii. If the agent had blocked the fire exits in the ordinary course of business, then the
partnership is liable.
1. Merely because something is illegal doesn’t mean it’s not in the ordinary
course of business.
iii. Standard for what’s in the ordinary course of business for a partnership:
1. The kind of thing that a partner would do.
2. Occurred substantially within the authorized time and location of the
partnership.
3. Motivated at least in part by desire to serve the partnership.
iv. You’re personally liable for a partner’s intentional tort if:
1. It’s in the ordinary course of business; OR
2. You consent.

F. Liability in Partnership
a. Chief disadvantage of partnership is that you’re liable for all assets of the partnership.
b. Joint and Several Liability: Any Δ can be sued for the whole amount of the judgment.
c. Joint Liability: You must join all the parties in the same case. All the obligors are
indispensable parties to your action.
d. Enforcing Partnership Liabilities Against Partners
i. Head v. Henry Tyler Construction Corp.
1. This case changes the Alabama standard to joint and several liability for
partnership under the law of contracts.
a. Under the old UPA, you did not have to exhaust remedies
against the partnership.
b. You couldn’t even sue a partnership under the old UPA because
of the aggregate theory of partnership (as opposed to the entity
theory).

22
c. However, many states have changed their laws of civil procedure
to allow partnerships to sue and be sued.
2. The RUPA provides that liability is joint and several, but you must
exhaust remedies against the partnership first.
3. Even though you don’t have to, you should join the partnership and all
the partners in the same case.
a. You don’t have to wait to sue the partners. Exhausting remedies
against the partnership just means collecting against the
partnership first.

G. Ownership Interests and Transferability in Partnerships


a. General rule: No new partners without unanimous consent of existing partners. However,
transferring the right to receive income can be done unilaterally.
b. Kay v. Gitomer
i. Facts:
1. Lot 5 was conveyed to a partner. Issue: Was it partnership property?
ii. The standard is to look to the intent of the acquirer on acquiring the property.
iii. In this case, they bought the property after forming the partnership and filed their
tax returns declaring that the property was partnership property.
1. Without this, it’d be a very tough case to make that the property is of the
partnership.
iv. UPA §10(1) doesn’t apply because the property title wasn’t in the name of the
partnership.
v. UPA §10(4) applies because the property title was in one or more partner’s
name. So, the property conveyance was valid.
1. Because it’s not an ordinary business action, authority requires
unanimous consent of the partners (2 votes in this case).
vi. This case eliminated the difference between equitable and legal title.
vii. UPA §10(5) also applies because the title is in everyone’s name and it’s executed
by all the partners (Eckles had given authority for the transfer, therefore it was
executed by both partners).
1. Kay has authority because Eckles never complained about the
conveyance.
c. Rapoport v. 55 Perry Co. (NY)
i. The Rapoports wanted to make their children full partners with full rights.
ii. Rules:
1. However, absent a contractual modification, it takes unanimous consent
to make a new partner.
a. Policy: Partners are fully liable for the actions of other partners.
You shouldn’t be forced to be liable for the actions of another
without your consent.

23
2. You can transfer a right to partnership income unilaterally. The
transferee will only have an interest in the profits – no control over
management.
iii. The Rapoports argue that the default rule was changed in paragraph 12 of the
partnership agreement by requiring a majority (rather than unanimity) to make
new partners.
iv. The other partners argue that paragraph changed the default rule to make
transfers of income rights more difficult, requiring a majority vote rather than no
vote.
1. They might want to do this because giving away income rights might
disincentivize the transferor, who no longer gets a share of the profit if
the partnership is successful.

H. Creditors of partners
a. Hellman v. Anderson
i. A good rule of thumb is that whatever a partner can transfer, a creditor can seize.
ii. RUPA § 504 says you can get a charging order to make a partnership pay
creditors instead of the partner.
1. The creditor has no other rights other than to collect checks.
iii. A debtor partner has a right to reclaim his share of the partnership profits by
paying off the creditor.
iv. Foreclosure:
1. On foreclosure, that partnership interest goes to the buyer and the debtor
partner loses it forever.
2. A creditor/foreclosure can petition the court to dissolve the partnership,
causing distribution of the partnership assets (including those going to
the transferee).
a. It’s the last resort if the charging order doesn’t yield a good
stream of income.
3. The court will decide whether it’s equitable to foreclose or dissolve the
partnership.
a. They’ll look at whether the partnership is being unfair by
withholding payments to the creditor.
b. Also, whether it would disrupt the partnership’s business.

I. Duties Partners Owe Each Other


a. Meinhard v. Salmon (New York)
i. Facts:
1. M and S’s original deal was to upgrade property and turn it into shops
and such. Their partnership had a 20-year term.
2. S is the managing partner. M will get 40% of profit for 5 years, then 50%
for the rest of the term. Losses were to be shared equally.
3. Gerry, who owns the reversion interest in the property, approached
Salmon and gave a proposal to Salmon for a lucrative business
opportunity.
a. Salmon pursued it without telling Meinhard.

24
ii. Holding: “Partners owe one another a punctilio of honor the most sensitive.”
1. Meinhard changed all the major default rules of partnership.
2. The Court said Salmon owed a fiduciary duty to Meinhard to tell him
about the opportunity.
3. Cardozo makes it sound like partners have a classic fiduciary
relationship, where the fiduciary must be selfless. This isn’t correct.
a. This has to be wrong because partners have legitimate selfish
interests (to make money, to enforce contracts against other
partners, etc.) within the partnership.
4. Under RUPA, partners have a duty of care and loyalty, not selflessness.
5. Cardozo said Salmon denied Meinhard legitimate opportunity to
compete.
a. RUPA has a duty to disclose, but it’s not as intensive as what
Cardozo prescribes.
6. Why would it have not been a breach of duty to seize the opportunity
without Meinhard (Salmon’s arguments)?
a. Because the partnership will have come to an end by then.
b. One might argue, though, that the point isn’t to allow Salmon to
detach from Meinhard, but rather to allow them both to get out
of the current venture.
7. The benefit to the fiduciary rule is that it protects minority partners.
a. Downside: It reduces flexibility in management.
8. The law has generally followed Meinhard, but not to the extent imagined
by Cardozo. This much is true: Salmon should have disclosed.
b. Singer v. Singer
i. Two members of a family partnership formed their own partnership and bought
land unbeknownst to the rest of the family.
ii. It was an opportunity the partnership wanted to pursue – under RUPA 404, their
action would be a breach of the duty of loyalty.
1. However, the Singers’ contract adjusted the default rules to allow for
competition of that nature.
iii. RUPA 103(b) is a list of partnership rules that can’t be waived in contract
(regulatory rules).
1. You can’t eliminate the duty of loyalty, but you can define what the duty
is as long as it’s not “manifestly unreasonable.”
iv. The Singers’ contract wouldn’t be enforced under RUPA because it isn’t
reasonable.

J. Self-Dealing in the Partnership Context


c. RUPA 103: Partners can agree to allow self-dealing as long as it’s not manifestly
unreasonable.
d. Enea v. Superior Court (California)
i. The partner is upset because the Daniels rented partnership property to
themselves at a below-market rate (clearly a breach of the duty of loyalty).
ii. What if the Daniels had rented to themselves at a market rate?

25
1. Still non-permissible. Under RUPA 404(b), a partner can’t take an
adverse position from the partnership (as in a lessor-lessee situation).
2. HOWEVER, RUPA 103 would allow an agreement to adjust the duty of
loyalty permitting self-dealing as long as it’s not manifestly
unreasonable.
3. RUPA 404(e) says something isn’t a violation just because a partner
does something in her own interest.
4. RUPA 404(f) says you can loan money and transact business with the
partnership at arms’ length.
iii. How to reconcile RUPA 404(b) with RUPA(e) and (f)?
1. As long as the partner is paying fair market value, the partnership still
benefits.
2. 404(e) and (f) ensure that a partner can enforce a contract against the
partnership in an arms-length transaction.
a. In the case of a legitimate contract, there’s nothing wrong with
enforcing it like an outsider.
3. (e) and (f) are exceptions to (b) that come into play when there is an
arms-length transaction authorized under RUPA 103.
4. If the partners agree to allow an arms-length contract, it must be
enforceable, and (e) and (f) make it permissible to deal with the
partnership as an outsider would.

K. Duty of Disclosure
e. RUPA 403 says a partner must inform other partners of relevant information for the
business without being asked.
f. Walter v. Holiday Inns
i. A tale as old as time: when a partner buys out another partner, the buyer will
complain if the business does poorly and the seller will complain if the business
does well.
ii. A partner only has to disclose information that’s material.
1. Material information is information that a reasonable investor
would want to know.
iii. A standard like that looks different with hindsight. The standard is supposed to
only look at the investor without the benefit of hindsight.
iv. Reliance is not necessary. The reasonable investor must have only wanted to
consider the information for it to be material.
1. It’s not necessary to prove reliance in a securities fraud case, either.
v. Do partners have a fiduciary duty to each other when there’s a buyout in the
offing?
1. At common law, a fraud is either:
a. A misrepresentation,
b. A half-truth, OR
c. Silence (when there’s a duty to make a disclosure).
2. During a buyout, there’s an adverse interest, and both parties know it.
Therefore, there’s only a duty to avoid misrepresentations and half-
truths.

26
a. However, by statute in New Jersey, partners have a duty to
disclose even during buyouts.
vi. SCOTUS said there’s no distinction between reliable and unreliable material
information. If it’s material, it must be disclosed.
vii. The court here, however, says Holiday’s projections weren’t material because the
πs had access to all of Holiday Inn’s data.
1. Ragazzo says this is wrong because πs aren’t able to make the same
projections as Holiday Inn with its greater capacity, etc.
2. However, for cashflow data, there’s no particular expertise needed.
viii. Holiday also withheld a report on how the casino could be fixed and made
profitable (which Holiday later used to make the business profitable).
1. However, it wasn’t finished at the time of the buyout. And the Holiday
management didn’t know about it.
ix. Whether securities law applies to a transaction depends on whether the
partnership interest takes the form of an investment contract.
1. To be a security, there must be:
a. An investment;
b. Common enterprise;
c. Profits expected to come solely from efforts of others.

L. More Duty of Loyalty


g. Meehan v. Shaughnessy
i. When is leaving a partnership to start your own partnership a breach?
1. It’s perfectly fine not to tell anyone until the moment you leave, as long
as you do everything else right.
2. It’s also fine to begin the logistics of the move.
3. You can’t, however, compete for employees on a secret basis.
a. Stealing employees while you’re still a fiduciary is obviously
illegal.
4. You also can’t approach clients secretly.
a. You have to tell the partnership you’re leaving, then give the
clients a chance to choose whether to stay or go.
5. You can’t intentionally delay cases to save billings for your new
partnership (though you can plan to do it then not do it)
ii. MBC (the breakaway partners) did this wrong by approaching the clients in
secret, jumping the gun on giving notice to the clients, and not giving the clients
a chance to choose whether to stay or go.
iii. The court says MBC must turn over the firm’s share of all revenue derived from
the poached clients.
1. Why not allow MBC to deduct the cost of MBC’s labor?
a. Because the value of a lawyer’s labor is hard to quantify, they
just say the labor is worth 0.
i. The court may also be punishing MBC.
2. MBC has the burden to prove lack of causation (i.e. – that the clients
would have left with or without the breach of loyalty).

27
h. Bane v. Ferguson
i. There’s no duty to former partners.
ii. In this case, a former partner of a law firm had no cause of action for breach of
fiduciary duty against his former partners for making an unwise decision that
resulted in the termination of his pension.
1. The court found that because he was no longer a partner, the other
partners didn’t owe him any duties.

M. Leaving a Partnership
i. Note on Accounting:
i. At common law, an accounting is the only way a partner can sue another partner,
and it can only occur at dissolution.
ii. UPA: Still only an accounting, but doesn’t have to accompany a dissolution.
iii. RUPA: You can sue a partner for anything. Also, a partnership can sue and be
sued (because of the entity theory).

j. Page v. Page
i. One brother wants to leave the partnership, but the other brother says he can’t.
ii. Rule: A partner can leave at will unless the partnership is for a term or an
undertaking.
iii. Brother 2 says it was a partnership for an undertaking.
1. However, there’s no written agreement. They orally said, “The business
will stay around until it pays for itself.”
iv. What they seem to have agreed to is that business profits will go toward paying
off the loan.
1. Therefore, this partnership is terminable at will.
v. The buried body: The Air Force Base opening up next door was going to make
the business very profitable.
1. So, the remaining brother was concerned that the leaving brother was
going to steal the business.
2. Partners owe each other a “punctilio of honor most sincere.”

k. Bohatch v. Butler & Binion


i. Bohatch turns her partner in to the client for overbilling.
1. When they go to the client, the in-house counsel says there’s no problem.
a. In-house counsel was probably in on it.
ii. So, the other partners all but fire her.
1. There’s nothing wrong with kicking out a partner as long as it’s not for a
bad reason.
2. Nobody has to be partners if they don’t want to be partners.
iii. Would it be OK to throw out a partner for the purpose of stealing the business?
1. No. Meinhard and Page say this is illegal.
iv. The remedy for wrongfully getting forced out of a partnership is the present
value of lost wages.
1. In this case: Present value of the difference between wages at Butler &
Binion and her other work.

28
v. The trial court awarded $57,000 for past lost wages, $250,000 for mental
anguish, and $4 million in punitive damages.
1. Why no lost wages?
a. She probably has a contract that says she only gets what’s in her
capital account when she leaves.
b. You can contract around the buyout requirement under RUPA.
So, whether she was thrown out rightfully or wrongfully, she
gets only what’s in her capital account.
c. The B&B partnership agreement also said the partnership
wouldn’t dissolve when a partner leaves.
2. It has to be right that you can’t be forced to be a partner with someone
you don’t like.
a. Therefore, no mental anguish and no punitive damages.

N. Dissolution of the Partnership


l. Dreifuerst v. Dreifuerst
i. UPA § 38 says partners get paid in cash on the winding up of the partnership.
The partners who want to stay in business can’t keep the business assets – they
must be liquidated.
ii. The leaving partner in this case wants cash, while the remaining partners want to
keep the mills they run.
iii. Problems with in-kind distributions:
1. The mills probably sell for more as a unit rather than piecemeal.
2. The mills are difficult to valuate without a sale.
3. Need cash to pay off the partnership’s creditors.
iv. In circumstances where courts think it’s unfair to force a sale, they will often do
an end run around the cash-only provision.
v. RUPA also allows a partner leaving at will to dissolve the partnership.
vi. The Texas rule doesn’t dissolve the partnership when a partner leaves at will.

m. Drashner v. Sorenson
i. Drashner complains that the partnership isn’t paying him enough money to live
on. However, the partnership agreement doesn’t allow for more than what he’s
getting.
ii. The partnership must be dissolved because Drashner’s actions made it
impracticable to carry on the business.
iii. Rule: The consequence of wrongful dissolution is that the person who caused it
gets his allotted distribution, minus good will.
1. This is a substantial punishment, particularly in service businesses.
iv. RUPA 701(a): If you wrongfully dissociate yourself, then RUPA says you must
compensate the partnership for damages, then you get your share of the
partnership.
1. You don’t get the good will deduction, as with the UPA.

n. Saint Alphonsus v. MRI Associates


i. Why isn’t Saint Alphonsus allowed to leave?

29
1. It’s not one of the four reasons given for dissociating in the contract.
ii. Rule: It has to be exceptionally clear in the contract that you can’t leave the
partnership without damages.
1. The tradition that partners can leave at will is very strong.

30
III. Corporations
A. Overview
a. Main benefit: Limited liability
i. Individuals aren’t personally liable for the obligations of the corporation.
ii. It’s a form of social welfare – society bears the costs which can’t be charged to
the shareholders.
b. The law of corporations is generally created with public companies in mind.
c. Characteristics of a Corporation:
i. Corporations are entities and are perpetual.
1. You can’t force a corporation’s dissolution if you’re a minority
shareholder.
2. It’s much harder to get out if there’s not a ready market for your stock.
a. For IBM’s shareholders, there’s no problem.
ii. Corporations are centrally controlled.
1. There’s a board of directors to whom the shareholders defer control.
a. Compare to a partnership, where each partner usually does the
business.
iii. Shares of a corporation are personal property and are freely transferable.
1. Compare to a partnership where unanimous consent is required to
transfer your partnership interest.
d. You must choose a state of incorporation – you must file a certificate with the secretary
of state.
i. Corporations, unlike partnerships, can’t be formed by accident.
ii. Most corporations incorporate in Delaware.
1. DE has an enabling, rather than regulatory, philosophy. Rather than
focusing on protecting shareholders and society from corporations, DE
sees its mission as providing for maximum flexibility.
2. Because so many corporations are headquartered in Delaware, there is a
large corpus of corporate law – results are more certain.
3. If there’s no law on a subject in your jurisdiction, you look to DE law.
4. DE has a deep, knowledgeable bench and a talented bar of corporate
attorneys. The DE Supreme Court is the most important state court in
business law.
iii. For smaller companies, DE law isn’t that different from any other state’s law.
Therefore, smaller companies are better off incorporating where they are located
(so they won’t have to pay two sets of incorporation fees).
e. You can finance a corporation however you want, within the limits of DGCL § 151.
i. You have to list the types of shares in the certificate.
f. Preemptive rights:
i. The right to first dibbs on newly issued shares.
ii. Putting out preemptive rights dilutes the value of existing shares.
iii. In most states, the availability of preemptive rights must be established in the
certificate of incorporation.

31
J. How to Incorporate in Delaware
g. File a certificate of incorporation.
i. The certificate must have a laundry list of information required by DGCL § 102:
1. Name, including a word like “association,” “company,” “corporation,” et
al.
2. Address
3. Nature of business
4. Number of shares of each class of stock, along with the rights and
powers associated with each class.
5. Name and address of the incorporators or, if they’re stepping down once
incorporated, the directors who will serve until the first annual meeting
of stockholders.
ii. As soon as the certificate is filed, the corporation is active if it’s statutorily
correct.

K. Binding Corporations in Contracts


h. Promoter’s Contracts
i. A promoter looks for people to buy shares in a yet-to-be formed corporation.
They allow a corporation to hit the ground running.
ii. Issues: Did the 3rd party have a contract with the promoter before the corp’s
formation, or did the promoter merely extend an offer on behalf of the
corporation?
1. When the corporation comes into being, did it become party to the
promoter’s contract by adopting it? (Ratification isn’t possible because
the corporation didn’t exist yet, but mostly it’s the same concept.)
a. Adoption can be express or implied
b. Difference: Ratification travels back in time and the principal is
retroactively bound. In adoption, the corporation is bound as of
the date of the adoption.
2. Another issue: Did the corporation substitute itself for the promoter in
the contract, or is the corporation added as an additional party?
i. Jacobson v. Stern
i. Stern was doing work on the corporation’s property before the corporation came
into existence, meaning that Stern’s contract was with Jacobson.
ii. When the corporation comes into being, it adopts the contract by accepting the
benefits of Stern’s work and signing his checks.
iii. Was there a novation (substituting Jacobson with the corporation in the
contract)?
1. Under the ordinary law of contract, yes, but under corporate law, it must
be clear that the parties agreed to a novation.
a. Therefore, Jacobson is still liable on the contract along with the
corporation.
b. Jacobson should have gotten Stern to sign something agreeing to
a novation releasing the promoter.

32
i. Always be sure the third party signs such a thing or the
promoter will remain liable on the contract.

L. De Facto Corporation / Corporation by Estoppel (Delaware)


j. Elements of De Facto Corporation:
i. Allowed in incorporate.
ii. Good faith attempt to incorporate.
iii. Purported to act as a corporation.
k. Corporation by Estoppel Doctrines
i. A corporation is estopped from using its own improper incorporation as a
defense.
ii. Third parties can’t use the corporation’s non-existence to get out of a contract
with the corporation.
iii. Shareholders retain limited liability in estoppel situations.
iv. Only applies in contract cases. A plaintiff in a tort case isn’t estopped from
arguing that a taxi company isn’t a corporation.
l. Cantor v. Sunshine Greenery
i. If you get everything right in your incorporation, you’re a de jure corporation.
ii. If you come close, you may be a de facto corporation (a corporation as against all
the world except for the state).
1. An attorney general can bring a quo warrento action to challenge an
incorporation.
iii. Sunshine mailed the articles of incorporation to the right address, but they got the
secretary of state’s name wrong.
1. This was most likely a good faith attempt.
iv. Sunshine can’t get out of their contract even if they weren’t a corporation at the
time of the contract.
1. They are bound by corporation by estoppel.

M. De Facto Corporation / Corporation by Estoppel (Model Act)


m. Robertson v. Levy
i. Timeline of events:
1. 12/27/61: Articles of incorporation mailed
2. 12/31/61: Seller assigns lease to corporation
3. 1/2/62: Secretary of State rejects the articles of incorporation
4. 1/8/62: Seller transferred assets to corporation in exchange for a note
5. 1/17/62: Articles of incorporation accepted
ii. Neither de facto corporation nor corporation by estoppel help the shareholder in
this case. It didn’t matter that the seller thought he was dealing with a
corporation.
iii. The Model Act doesn’t accept the third branch of corporation by estoppel or de
facto corporation doctrine.

33
1. The Model Act makes it clear there’s only one point in time when there
is limited liability: Once the Secretary of State accepts the filing or issues
a certificate, it constitutes proof against all the world that the corporation
was properly formed (except for the AG – he can still quo warrento you).
n. Even in Model Act jurisdictions, the first two branches of De Facto Corporation and
Corporation by Estoppel apply.

N. Ultra Vires Doctrine


o. A corporation’s board is limited to the actions envisioned in purpose section of the
articles of incorporation. However, the purpose can be very broad (e.g. – “Any legal
activity.”)
p. Goodman v. Ladd Estate Co.
i. Ladd Estate won’t guarantee a shareholder’s loan unless his corporation backs
him, therefore the corporation backs the director’s loan.
ii. The purpose of the corporation didn’t contemplate guaranteeing the director’s
debt.
iii. The only good reason to put in a purpose clause is to bind the hands of a board of
directors in a closely held corporation.
iv. The loan guarantee was ultra vires because the loan was for the director’s
personal benefit.
1. This has always been ultra vires and remains so today.
v. Even though it’s ultra vires, the corporation can’t get out of the loan guarantee.
1. DGCL § 124 says that even if a contract is ultra vires, only certain
people can enjoin the performance of an ultra vires contract: shareholders
who didn’t vote for the ultra vires action or the attorney general.
a. To challenge the ultra vires contract, the shareholder must show
that (1) every party in the suit is a party to the challenged
contract, (2) the contract is executory, and (3) the relief is
equitable.
i. In short, it’s almost impossible for a shareholder to
challenge an ultra vires contract.
ii. Allowing too many ultra vires challenges would hamper
directors’ management authority and create uncertainty.
b. Shareholders who voted for the ultra vires action can’t challenge
it. If you got your shares from a shareholder who approved the
action, then your shares are tainted, and you can’t challenge
either.
q. Dodge v. Ford Motor Co.
i. Shareholders are upset because Henry Ford decided not to give out dividends, but
rather to reinvest in the business – thereby benefitting society as a whole.
1. Ford wanted to lower the prices of his cars and hire as many workers as
possible.
ii. Court: The corporation’s sole purpose is to make money for the shareholders.

34
iii. There’d be no problem if Ford had just lied about why he was reinvesting profits
– some business-related reason like increasing market share, etc.
1. Ford lost because he told the truth. Because of the business judgment
rule, any decision of the board of directors must stand as long as there
was a legitimate business purpose.
r. Policy: We’ve long since given up on the 19th century view that corporations are limited
in their purposes.
i. They’re where the money is. They have a huge role in this country. Perhaps they
should be allowed to help society.
ii. Should a corporation be allowed to make an anonymous donation?
1. A tenuous argument may be that benefitting society indirectly benefits
shareholders.
iii. Can a corporation take an action for the benefit of society – with no benefit to the
shareholders?
1. AP Smith v. Barlow (New Jersey) says that a corporation does have the
right to benefit society at no benefit to shareholders.
2. As a matter of symmetry – because corporations benefit from society
(indeed, can’t exist without society) – corporations should be allowed to
give back to society to repay society’s largesse.
iv. Generally, the law is that it’s OK for corporations to act to benefit society as long
as the contribution is reasonable in amount (relative to the size of the
corporation).
v. Revlon said corporations can act for the benefit of various constituencies beyond
the shareholders as long as there are rationally related benefits accruing to the
shareholders.

O. Management and Operation: Allocation of Power


s. Charlestown Boot & Shoe Co. v. Dunsmore
i. The shareholders wanted Dunsmore to let Osgood help them wind up the
corporation. Dunsmore made a hash of the wind-up and lost thousands of dollars.
ii. Under DGCL § 141(a), the shareholders lose. The board runs the corporation
however they want – they don’t have to get Osgood’s help.
iii. DGCL § 242 says it takes a resolution by the board of directors to amend the
corporation’s certificate.
1. This would not work in Charlestown Boot because the board consists of
Dunsmore, who will not cooperate with the shareholders.
iv. DGCL § 211(d) says the board must call a stockholders meeting – this is
necessary before amending bylaws.
v. DGCL § 228 says written consent can serve in the place of a stockholder
meeting.
vi. DGCL § 109 says stockholders have the power to amend the bylaws once money
has been given to the corporation for its stock.
vii. DGCL § 223 says the board fills empty board seats.
1. This is also not helpful for the Charlestown Boot shareholders.

35
viii. So, the shareholders should submit written consent in lieu of a meeting, amend
the bylaws to say there must be an additional seat on the board, that shareholders
must fill the empty seat, and that Osgood will be the new director.
1. This is good corporate lawyering.
2. However, none of this would work if the certificate laid out the rules for
electing the board of directors (only board members can amend the
certificate). Also, if the certificate said written consents aren’t allowed,
then the shareholders are screwed.
t. Blasius: The board can’t interfere with the shareholder franchise without a compelling
justification. An example of a compelling justification would be, “The shareholders need
more information, so we must delay the election.”
u. The bylaws of a corporation can have required qualifications for board members (DGCL
§ 141(b)).
i. Stroud v. Grace
1. In this case, the board amended the bylaws to make qualifications:
a. They must be outsiders with substantial business experience (so
that there is disinterested supervision).
b. There must be at least two shareholders on the board (so the SHs
are represented).
c. No more than two current officers can be on the board.
i. It’d be inappropriate to have too many officers because
the board is supposed to supervise the management, and
putting too much management on the board will
undermine the supervision.
2. The Strouds allege that the Millikins are planning ahead to when they
won’t have enough shares to vote themselves in. So, they planned these
qualifications so their board members will be the only ones qualified.
a. The Strouds can’t put a qualified slate together because of the
first qualification. The Strouds aren’t running the business like
the Millikins, so they lack the connections with qualified
shareholders that the Millikins have.
3. Are the qualifications legal?
a. The court says Blasisus isn’t implicated because the main
purpose of the amendments was not to interfere with the
shareholder franchise.
i. Courts only apply Blasius when the board interferes with
an imminent shareholder vote.
b. Even if Blasius did apply, there’s compelling justification in
improving corporate governance. Also, a majority of the
shareholders voted for it.
4. The Strouds were also upset that the bylaws let the board decide on
whether candidates are qualified, including the “substantial business
experience” requirement, which is a nebulous concept.
a. The court said this is only a hypothetical problem – it’s facially
valid, so there can only be an as-applied challenge.

P. Quorum Rules:

36
v. Gearing v. Kelly
i. If there are 9 directors, then a quorum is 5. A quorum is a majority of authorized
board seats.
ii. DGCL § 141(b): The certificate or bylaws can require a greater number for the
quorum to be met. Unless the certificate says otherwise, the bylaws can require a
smaller quorum, but it can’t be less than 1/3 the total number of directors.
iii. In Gearing, the Gearings and Kellys have a company which they are unwinding.
To prevent an election to fill an empty board seat (the fourth one), Mrs.
Meacham just stays home to keep the meeting below quorum.
1. She’s the third director, so quorum is 3 (a majority of 4).
iv. The Court says directors have a duty to attend directors’ meetings, and Mrs. M
breached that duty. So, she’s estopped from seeking a new election.
v. To give better protection for minority shareholders, you should require a
supermajority in voting.

Q. Authority in the Corporate Context


w. Lee v. Jenkins
i. Lee says Mr. Yardley promised him a full pension regardless of whether he was
65 when he left the Jenkins’ corporation.
ii. There are statute of frauds and Dead Man’s Statute issues at play, but the real
problem is the court thinks Lee’s assertion isn’t credible.
1. The Court said what probably happened was that Yardley promised to
honor Lee’s previous pension rights, not that he’d have better pension
rights. Lee’s version of the promise would give him pension rights better
than anybody else’s coming from the old company.
iii. Authority Issue: Is Jenkins bound by Mr. Yardley’s promise?
1. Learned Hand says no.
a. No express actual authority because nothing in the certificate,
bylaws, or the board’s resolutions said Yardley could make such
a promise.
b. No implied actual authority because a reasonable person in Mr.
Yardley’s position wouldn’t have thought he had this power.
c. No apparent authority because there’s no course of conduct Lee
is aware of.
2. Why isn’t the analysis for implied authority the same as for apparent
authority?
a. When authority flows from the power of position, it should be
the same question unless there’s some insider communication
that a third party wouldn’t be privy to.
3. Presidents have authority to take ordinary action, but not extraordinary
action.
a. E.g. – Regular hiring is ordinary. Extraordinary hiring would be
giving someone a life-time contract.
i. A life-time contract is extraordinary because of the
length of time and the amount of money invested.
4. Is a pension an extraordinary action?

37
a. The amount isn’t a big deal compared to Yardley’s other powers.
b. However, it is forever, so it’s a large time commitment.
5. Delaware Rule: Apparent authority is limited to what a reasonable person
would believe that a president can ordinarily do.
6. Texas Rule: No authority flows from the position alone.
7. All state agree that the secretary of a corporation has the authority to
certify corporate records.

R. Shareholder Voting
x. DGCL § 216: A quorum of shareholders is met with a majority of the shares entitled to
vote, either in person or by proxy.
i. It takes a majority of the quorum to pass something.
ii. Abstentions are actually a “no” vote, because it still requires a majority of the
quorum.
y. Ordinary versus Extraordinary:
i. It requires a majority of all shareholders to approve an extraordinary action.
ii. For ordinary matters, just a majority of the quorum.
z. Types of voting:
i. Straight voting shares allow a majority to elect the entire board. You vote the
same number of votes for all the seats.
ii. In cumulative voting, multiply your votes by the number of seats.
1. Then, you split the votes amongst the candidates in any way you like.
2. Formula: N= (X(D+1))/S
3. X=shares you own; D=number of directors; S=total number of shares;
N=number of directors you can elect with your shares.
4. *You have to round down to the nearest whole number.
aa. Cumulative voting is more proportional for minority shareholders.
bb. In straight voting, a minority gets nobody elected.
cc. Why does it matter? A minority SH would have more access to information if they have a
director on the board. Directors have a right to see all corporate information; SHs don’t.
dd. In Delaware, straight voting is the default rule. In Texas, corporations established after
2003 have straight voting as the default rule.
ee. In a cumulative voting corporation, you can only remove an individual director if the
votes cast against a director’s removal would have been enough to elect him.
i. If the entire board is removed, this rule doesn’t apply. This is so a majority can’t
undo the effects of cumulative voting.

ff. Shareholders’ Information Rights


i. Skouras v. Admiralty Enterprises, Inc.
1. Rule One: A shareholder can only get corporate information if he has a
proper purpose. The corporation has the burden of showing the
shareholder doesn’t have a proper purpose.
a. Why can’t shareholders see the books and records all the time?
i. The potential for harassment and the inefficiency
involved.

38
b. If the shareholder’s proper purpose is to discover corporate
wrongdoing, then he must have some evidence of it first.
2. The π in this case says the corporate board and officers are wasting
money and embezzling.
3. The Δs say the π is just harassing the Δs to get them to buy him out.
4. What if I am a shareholder for a corporation that sells drones that kill
people in the Middle East and I want a list of other shareholders?
a. You’re not going to convince the shareholders to change the
purpose of the drones.
b. So, the real reason is probably to publicize the names of the
shareholders of the evil company.
ii. Per Credit Bureau Reports, communicating with other shareholders is always a
proper purpose and it doesn’t matter if there are ulterior motives for getting the
list of shareholders.

S. How Shareholders Can Order Their Affairs Through Contract


a. Any default rule can be modified by contract, while regulatory rules cannot be modified.
b. Ringling Bros. v. Ringling
iii. Shareholders had a vote pooling agreement. If they didn’t agree on something,
Mr. Loos would be their mediator.
iv. If they voted separately, they could each elect two directors, but if they pooled
their votes, they could elect 5. Each owned 315 out of 1000 shares.
1. N= (315*(7+1))/1000 = 2520/1000 = 2.52
v. At the election, the two deal-makers don’t agree on how to vote, so there was an
adjournment. This adjournment should have passed because, by the voting
contract, there’d be 630 votes for it.
1. However, the contract was not self-enforcing. The chairman doesn’t care
about the contract, only about who is allowed to vote per the certificate
and bylaws.
2. It’s not the job of the chairman to enforce contracts. Mrs. H breached her
contract, but that’s not the corporation’s problem, it’s the courts’
problem.
vi. So, they go to court, which says the voting contract was legal.
1. The court says Mrs. H’s votes don’t count because she breached the
contract.
2. The problem with this is that Mrs. Ringling didn’t get what she
bargained for: 5 out of 7 directors.
vii. Is this the real purpose of the agreement?
1. Separately, they could already elect 4 out of 7 directors. Why is it better
to have 5 of 7?
2. The agreement is probably really about votes that aren’t for director. By
pooling their votes, they now control 100% of the non-director votes.
viii. DGCL § 218 permits this arrangement. This type of deal is legal because it
decreases the risk of investment – thus, corporations have to pay out less and can
gather capital more cheaply.

39
1. The only kind of voting agreement that’s illegal is a vote-buying
agreement.
ix. DGCL § 212 allows for proxy votes. It only requires that the agreement be in
writing and signed by the vote-holder.
x. Problems with voting contracts:
1. You have to go to court to get them enforced.
2. Courts make mistakes, as in this case.
xi. Could they have averted this problem by making Mr. Loos a proxy?
1. Only if the proxy is irrevocable. DGCL § 212(e): The proxy agreement
must be (1) in writing, (2) signed by the parties, (3) state that it’s
irrevocable, and (4) the proxy must be coupled with an interest.
a. The interest can be an interest in the stock or an interest in the
corporation itself.
b. A CEO/director is someone who has an interest in the
corporation (their careers are attached to the corporation’s
success, giving them an interest that extends beyond the share
price).
2. Mr. Loos had neither an interest in the stock nor in the corporation, so he
can’t be an irrevocable proxy. Under the Model Act, an irrevocable
proxy can be made by contract, but Mr. Loos isn’t a party to the
shareholders’ contract.
c. Another option would be a voting trust.
xii. Mr. Loos would be a trustee. They’d make sure he voted the right way with a
trust agreement.
xiii. The trust agreement would be public, and you’d really have to trust Mr. Loos –
he could violate the agreement and the only recourse would be through the
courts.
xiv. Abercrombie says that if the substance of an agreement is that of a voting trust,
the court would enforce it as a voting trust though this may not have been the
parties’ intention.
1. DGCL § 218(b) overrules this, but Abercrombie’s rule applies in other
states.
d. Classes of stock can also solve the Ringling problem.
xv. You can have different classes have different voting rights attached.
xvi. Have Mrs. R take Class A stock, which allows election of 3 directors.
xvii. Have Mrs. H take Class B, which allows election of 2 directors.
xviii. Mr. N gets Class C, which allows election of 2 directors.
xix. In all other respects, the classes are equal.
xx. This must be done in a formal, mechanistic way, and carefully to achieve the
right result.
1. How to create a tiebreaker:
a. Make a class with no economic or voting rights other than to
elect one director.
e. McQuade v. Stoneham
i. The three directors have an agreement to keep the current three directors in
their current positions.
1. This is a legal agreement per DGCL § 218 and Ringling.

40
ii. The agreement says McQuade will be treasurer and have a salary of $7500.
iii. The agreement violates DGCL 141 in that it takes discretion away from the
directors in appointing officers and setting salaries.
2. Shareholders can be bound by voting agreements. Directors can’t.
3. There are other shareholders besides S, McG, and McQ. They can’t be
disenfranchised by the agreement.
iv. How to make sure McQ gets everything he wants:
4. DGCL 141 says the certificate can say McQ has to be treasurer and have
a certain salary.
b. DGCL 242(b)(1) gives the process for amending the certificate.
5. Another way to help McQ:
c. A self-enforcing voter agreement.
d. Also, make it so it requires a unanimous director’s vote to
remove officers.
e. The shareholder agreement makes McQ a director, so he needs
that to be enforceable.
f. Clark v. Dodge
v. In small, closely-held corporations, shareholders don’t care how their money
comes out. It can be salary or dividends and it’s all the same for them.
vi. In this case, an agreement that Clark would get ¼ of the profits and remain
treasurer was enforceable because all of the shareholders were also directors,
so there were no minority shareholders to protect.
vii. The agreement doesn’t run afoul of Long Park because it didn’t “completely
sterilize” the directors’ discretion.
6. Under no circumstances can you completely sterilize the board of
directors (Long Park).
a. In 2015, how do you get the Class A stock holder everything he wants?
viii. Amend the certificate to say the Class A stockholder can elect the board for 19
years.
b. Takeaway: The regulatory rules are really just traps for the unwary. There’s always a
way to arrange voting to get what you want.

T. Amending Bylaws and Certificates


a. Frankino v. Gleason
ix. Frankino wants control of day-to-day operations at the corporation. Why
doesn’t he already have it as a 55% shareholder?
7. There must be some contractual restriction on F’s ability to choose the
Board.
x. The company belonged to F before it went public, so he thinks he should still
be in charge.
8. This kind of conflict is inevitable when someone who’s used to being in
charge is a majority shareholder.
xi. To regain control, F wants to expand the Board from 6 seats to 13.
9. First Problem: He must get a majority of shareholders to change the size
of the board by amending the bylaws.

41
f. No problem there because he has 55% of shares and can hold a
meeting by written consent (DGCL § 228 allows this).
10. Second Problem: Article 3 of the bylaws can’t be amended without a
supermajority, per Art. 9.
g. Issue: Can Frankino amend Art. 9 with a simple majority?
i. Article 2 says a simple majority vote is all that’s needed
to resolve any issue unless otherwise provided for in the
bylaws.
ii. Answer: Yes. The super-majority requirement can be
modified with a simple majority.
h. DGCL § 242 says that if a corporation’s certificate requires a
super-majority to decide an issue, it takes a super-majority to
amend that certificate provision.
i. The certificate is a public, much more formal document
than the bylaws.
ii. The DGCL doesn’t require a super-majority to amend a
super-majority requirement in the bylaws.
i. The minority shareholders in this case could have fixed this issue
by putting something in the certificate.
i. It’s really bad lawyering that this wasn’t done in the first
place.
b. A supermajority voting provision is typically intended to protect minority shareholders
by giving them veto power.
xii. Nothing in life is free, though, so you have to pay more for your shares if they
come with veto power.

U. Transferability of Shares
a. Closely held companies often don’t like shares to be freely transferable. The existing
owners don’t like it when someone new shows up wanting to help run the business.
b. Share transfer restrictions are how you solve that issue. However, it must be done right.

c. Allen v. Biltmore Tissue Corp.


i. In this case, Mr. Allen wanted to sell his shares, but the corporation had a first
option pursuant to the bylaws.
11. But how to treat dividends?
j. He bought the shares initially at $5 per share, then got 5 shares at
$5 per share. Then 10 shares at $10 per share.
k. So, 10 shares at $5 basis and 10 shares at $10 basis. Average =
$7.50.
12. DGCL § 202 allows restrictions on share transfer as long as they’re
reasonable.
l. This anti-alienability rationale comes from the law of property –
there’s a policy objective of keeping property “freely alienable”
by its owner, or liquid and in commerce.
ii. Types of alienability restrictions:

42
13. The corporation’s first option agreement allows the corporation to buy
back Allen’s shares at the price he paid for them.
m. A requirement that the corporation buy back at FMV would
impose very high transaction costs for a small company.
i. Problems with valuation, dueling experts, etc.
ii. Also, the company might not have enough cash on hand
to buy the shares or know how much to set aside for that
purpose.
14. First refusal restrictions are routinely upheld to the courts but risky,
largely due to the risk of collusion to the corp. and chilling effect on
bidding for the SH.
15. A buy-sell agreement obligates the shareholder to sell, or the corporation
to buy, the shares under specified circumstances.
n. Such agreements are much less offensive to the law of property
(because it keeps property liquid), but from a shareholder
perspective it’s much worse because they might have to accept
or pay an unfair price. There’s no relation to FMV required.
16. Consent Restrictions: Consent to transfer of shares can’t be withheld
unreasonably.
o. Preserving tax or legal advantages, keeping a corporation in the
family, or preventing a competitor from acquiring shares are all
prima facie reasonable reasons to withhold consent.
iii. The property rules are mostly a trap for the unwary. You can get whatever you
want as long as the restriction on alienability isn’t unreasonable.

V. Statutory Close Corporations and Close Corporations


a. DGCL § 342–343 lays out the requirements for becoming a statutory close corporation.
b. A 2/3 vote of shareholders and the board is required to amend the certificate to say it’s a
statutory close corporation.
c. What’s the advantage of a statutory close corporation?
i. Shareholders can restrict the board’s discretion without running afoul of the
McQuade line of cases making it illegal to “sterilize” the board.
ii. The shareholders can make an agreement to recreate the rules of partnership,
including unilateral dissolution at will.
iii. Becoming a statutory close corporation exempts you from many regulatory
corporate rules.
d. Zion v. Kurtz
iv. Kurtz wanted to make a “bootstrap acquisition” of Lombard, meaning the assets
acquired secured the financing of the acquisition.
v. SEC regulations required Kurtz to have $4 million in net capital to act as a
broker/dealer, which he needs in order to acquire Lombard.
17. So, Kurtz went to Zion to get a guaranty which would put Kurtz over the
SEC asset requirement.
18. Zion wants a veto authority in exchange. Zion objects to the interest
requirement because it would increase the likelihood Kurtz will go under

43
and require Zion to make good on the guaranty. So, Zion wants to
exercise his veto on the proposition to have the note bear interest.
vi. Issue: Is Zion’s veto power legal?
19. It’s distinguished from Long Park because the shareholder power in that
case was affirmative, rather than negative.
p. (Long Park stands for the proposition that even if all SHs agree,
they can’t sterilize the board’s authority).
vii. Holding: The Court says the agreement giving the veto power doesn’t run afoul
of Long Park because they might have just amended the certificate with the same
people who made the agreement.
viii. Dissent: No. You need the formalities required for a statutory close corporation
to give adequate notice to new shareholders.
20. The dissent is probably right. Dicta in Nixon v. Blackwell (DE 1993) said
you need the statutory close corporation formalities to take advantage of
statutory close corporations’ exemption from McQuade, Long Park, etc.
e. Texas has a special provision allowing corporations to use contracts to do things a
statutory close corporation could do as long as all shareholders agree.

W. Piercing the Corporate Veil


a. Veil piercing happens when the corporation is correctly incorporated, but you do
something later to cause the veil to be pierced.
b. Why do we have limited liability in the first place? (See U. Chicago article page 412)
i. To limit the risk of investment.
ii. Also, to allow for diversification – you can hold a number of different
investments with minimal oversight.

c. Walkovsky v. Carlton
i. Carlton owns 10 taxi corporations, each running one or two cabs.
1. This was to limit the assets of each corporation.
ii. Mr. Walkovsky wants to do vertical and horizontal veil piercing – both to pool
the assets of each taxi company, and of Carlton himself.
1. Horizontal veil piercing holds sister companies liable for each others’
liabilities.
iii. C complied with the minimum liability insurance required by New York law for
each of his cab companies, so there is no veil piercing.
1. However, the dissent – and the later case – found the corporations were
inadequately capitalized for the inevitable liabilities that could arise.
iv. Why not police the capitalization of corporations with something like a State
Department of Adequate Capitalization?
1. The investigation costs would be huge.
2. Also, a lot of companies start up as undercapitalized – even in debt.
3. We don’t want to keep companies from getting off the ground on a
shoestring budget.
d. There is no firm test in veil piercing – it’s up to the caselaw and courts deciding what’s
fair or unfair.

44
i. Undercapitalization alone isn’t dispositive. Siphoning off corporate assets of an
undercapitalized corp tends to push it over the edge to fraudulent transfer which
calls for veil piercing.
ii. Elements of Fraudulent Transfer for a Corporation:
1. Inadequate value received by corp for corporate assets transferred out.
2. Unlikely there are enough assets left in the corporation relative to usual
liabilities.
iii. Why is it OK to start a company with inadequate capital, but not to take money
out of an inadequately capitalized corporation?
1. Nobody would dump the assets of a profitable company. It’s a clear
indication that the corporation is a sham. The strongest veil piercing case
is when the Δ siphons off assets from an inadequately capitalized
corporation.
iv. The problem with a fraudulent transfer claim is that it’s hard to prove.

e. Minton v. Cavaney
i. Veil piercing is OK in this case because:
1. They didn’t follow corporate formalities (no board meetings, no minutes,
no stock, etc.)
2. Inadequate capitalization.
ii. The veil piercing was deemed appropriate, but the π still lost because Cavaney
was not joined as a party in the original suit against his company. (Pennoyer
strikes: Cavaney can’t be bound by a case to which he was not a party).
iii. This case required both of the above factors.
iv. What if the corporation had followed all the appropriate formalities?
1. It wouldn’t have any more capital than if it hadn’t.
2. However, if you blatantly ignore formalities, it probably will never be
profitable and adequately capitalized.
f. There is a Texas statute that says failure to follow formalities isn’t a factor in piercing the
corporate veil.
i. Having said that, a corporate lawyer must ensure there is no question about
piercing the veil.
1. Having meetings, document the egress and ingress of assets in a record
book, have resolutions, etc.
ii. If a company becomes inadequately capitalized through normal business
activities, then there won’t be a veil problem.

g. Perpetual Real Estate Services v. Michaelson Properties, Inc. (Virginia)


i. Virginia’s test for veil piercing:
1. Undue domination and control by shareholders;
2. Corporate form used to obscure a legal wrong.
ii. The first element of the test is meaningless – all closely held corporations are
dominated by their shareholders.
iii. The second element just means the corporation was unfair to creditors.
iv. Every veil piercing test is basically the same – did the Δ unfairly use the
corporate form to foil creditors?

45
v. Despite all the veil piercing elements being present in this case, the court doesn’t
pierce the veil because this is a contract case.
1. The π knowingly contracted with a limited liability company. Π could
have made the shareholder personally liable in the contract, but did not.
They’re stuck with the bargain they made.
vi. Why the difference between a contract case and a tort case?
1. We expect contract parties to inspect the credit of the other parties.
2. See also KC Properties v. Lowell
h. Different approaches to veil piercing in contract cases:
i. Some states, like TX, say there is no veil piercing in contract cases unless there is
actual fraud.
ii. Middle states will find veil piercing in contract cases where the π is not expected
to investigate credit, etc. If the π is sophisticated, then there is no veil piercing.
i. Public companies and veil piercing
i. Normally involves parents and subsidiaries.
ii. Test:
1. Did the parent company exercise undue domination and control?
2. Did the parent respect the financial separateness of the companies?
iii. It’s OK for parent companies to oversee broad strategy of the subsidiary, but not
for the parent to run the day-to-day affairs of the subsidiary.
iv. Kodak won its case in Kodak despite the parent and subsidiary having the same
account because they had scrupulous record keeping to ensure separateness.
j. Don’t confuse veil piercing (derivative liability) with personal liability of the
shareholder/parent company/sister company.
i. Always be sure to ask a personal liability question before asking a veil piercing
question.
k. Reverse Veil Piercing: When the company becomes liable to the creditors of the
shareholder.
i. This isn’t normally an issue because if the corporation is valuable, then the
debtor’s stock is probably a sufficient remedy (stock can be seized)

46
X. Equitable Subordination
a. Elements:
i. Corporation must be undercapitalized.
ii. Shareholders withdrew money for their own benefit.

b. Costello v. Fazio
i. The bankruptcy trustee in this case is mad because the shareholders withdrew
their money from the company, then loaned it back (becoming creditors).
ii. The bankruptcy trustee wanted to subordinate the shareholders’ claims to those
of other creditors.
iii. The reasoning for the SHs’ actions was to get tax benefits and to equalize the
shareholders’ contributions.
1. Their claims should still be subordinated because, even with the innocent
motives, there was still a personal benefit.
iv. Was the shareholders’ withdrawal a fraudulent transfer? Yes.
1. They gave no consideration for the transfer.
2. There were likely not to have enough assets to pay debts.
v. However, the fraudulent transfer remedy is unnecessary.
1. Having the shareholders’ claims subordinated will achieve a better result.
The creditors will get the same amount of money, and the arrangements
amongst the SHs will be unchanged.
vi. There’s no basis for piercing the corporate veil, either, because the shareholders
were careful to keep the funds in the company so as not to hurt creditors (that’s
why they loaned the money back to the company).

Y. Duty of Officers and Directors


a. Officers and directors are agents of the corporation, and therefore owe the duties of care,
obedience, and loyalty.
i. There is also a duty to act lawfully.
1. There are times when it would be to the company’s benefit to act
illegally. In such a situation, shareholders can sue for damages resulting
from the illegal activity. However, any tangible profits to the company
are deducted from the damages owed to the shareholders.

b. Francis v. United Jersey Bank


i. Pritchard & Baird is a small reinsurer. Reinsurers take on part of the risk of an
existing policy held by the seeding company in exchange for a portion of the
premiums. A reinsurance broker sets up these deals and passes the premiums and
payouts between the seed and the reinsurer.
ii. Mrs. Pritchard’s sons are taking “loans” out of the company which they don’t
intend to repay.
1. This is definitely a fraudulent transfer, but the veil piercing remedy
wasn’t available because the sons already spent the money.
2. So, instead, they’re charged with conversion. Mrs. P could be sued for
aiding and abetting the conversion, but they’d have to prove she was at
least reckless in allowing it to happen.

47
iii. The best way to reach Mrs. P is to sue her for breach of her duty of care – this
only requires proof of negligence.
1. The standard for duty of care is: Did the officer/director act as a
reasonable person would have done under the circumstances.
2. It can’t be that a director can lose a case against creditors that she would
have won against shareholders – creditors step into the shoes of
shareholders in a lawsuit against a director. Normally, creditors can’t sue
a director for breach of duty.
iv. Mrs. Pritchard breached her duty by failing to attend board meetings and
reviewing the financial statements.
v. Why does Mrs. Pritchard owe this duty to creditors?
1. The company was a broker – it had a trust relationship with its clients.
a. Applies to banks and reinsurance, deal mainly with OPM
2. Also, directors owe a duty to creditor when the company is insolvent.
a. For these two reasons, the creditors are allowed to sue the
director for breach of duty.
3. It’s relevant that it’s a small closely held corporation because the
standard has to do with what a reasonable person would do under all the
circumstances.
vi. Mrs. Pritchard lost because she did absolutely nothing as part of her director
duties to stop the stealing.
1. If she had done anything at all, it’d be a tougher case.
vii. Causation is also an issue in this case.
1. There’s no way to know if the sons would have stopped stealing.
2. However, the court cuts the uncertainty against the wrongdoer to find
causation.
c. Directors’ Liability for Others’ Illegal Conduct
i. The DE court held in a case that a director of a public company isn’t liable for
officers’ or directors’ illegal conduct unless they failed to put reasonable
safeguards in place.

Z. The Business Judgment Rule


a. If the business judgment rule applies, a director/officer’s actions are presumed to be taken
in good faith belief that it’s in the business’s best interests.
b. Two elements of the business judgment rule:
i. Did not breach duty of care.
1. Adequate oversight/reasonable investigation
a. Standard in Del is gross negligence
ii. No conflict of interest w/ the corporation.

c. Shlensky v. Wrigley
i. It’s profitable to play baseball games at night because people with money work
during the day.
ii. Despite this, Wrigley’s decision to not install lights and only have the Cubs play
day games is protected by the business judgment rule.

48
1. The business justification is that night games could hurt the
neighborhood and drive down revenue somehow.
2. In other words, there’s a rational basis relationship of the Lee Optical
variety to a legitimate business purpose.
iii. There is no duty of care for substantive business decisions. All you have to do to
be protected by the business judgment rule is articulate a legitimate business
purpose – something anyone can always do.
iv. Why have the business judgment rule?
1. Business is inherently risky, and the policy is to avoid discouraging risk-
taking.
2. Also, you want risk because it’s possible to get higher returns.
3. It’s also directors’ statutory job to make such decisions (DGCL 141(a)).
v. The downside of the business judgment rule is it increases agency costs: the risk
that directors will act for their own benefit rather than the company’s.

AA. Duty of Care


a. The business judgment rule doesn’t apply unless there’s careful investigation and good
faith on the part of the director/officer.
b. Smith v. Van Gorkum
i. Van Gorkum approached Pritzker to meet about an offer to buy out the company
at a 45% premium ($55 per share total).
1. Pritzker negotiated a deal where he’d get the opportunity to buy 1.5
million shares at $38 per share if someone outbid him.
2. This is called a lockup option, and it makes it more expensive for a
competitor to outbid Pritzker.
3. There’s also a no-shop option preventing Van Gorkum’s company from
soliciting other offers.
ii. The Delaware standard is that directors/officers aren’t liable for ordinary
negligence, only gross negligence. (NJ and NY allow for liability based on
ordinary negligence).
iii. The directors accepted the offer without having a meeting or doing a study. The
court finds this to be a breach of the duty of care.
iv. Holding: The financial study is a bright-line requirement – it doesn’t matter what
the market price is, what the premium is, or what the directors’ idea of the value
is.
1. It didn’t matter that the shareholders had approved the sale because they
didn’t know that the directors hadn’t done a valuation study.
2. All it would take to get around this requirement is to find a friendly
investment banker to give a low valuation.
a. But meeting alone not enough
c. The DGCL partially abrogated Van Gorkum with DGCL 102(b)(7), which allows the
certificate of incorporation to disclaim directors’ liability for breach of the duty of care –
so, most Delaware companies have this disclaimed and Van Gorkum is mostly a dead
letter.

49
BB. Self-Dealing
a. DGCL 144: Whenever a corporation engages in a transaction with another
corporation/partnership/organization in which a director/officer of the corporation has a
financial interest, there is self-dealing (e.g. – a director of ABC Corp. selects his side
business as ABC Corp.’s principal supplier of parts).
i. Escape hatches. The self-dealing transaction is voidable unless:
1. The fact of the director’s self-dealing is disclosed to the board and is
approved by a majority of disinterested directors,
2. The fact of the director’s self-dealing is disclosed to the shareholders and
is approved by a majority, and/or
3. The transaction is fair to the corporation and has been approved or
ratified by the directors or shareholders.
ii. Disclosure means telling all material facts relevant to the self-dealing director’s
benefit and of the transaction itself.
iii. Fliegler v. Lawrence held that the second 144 escape hatch requires a majority of
disinterested shareholders.
iv. Effect of the escape hatches (Delaware):
1. Using hatch 1 or hatch 2 shifts the burden such that the π must prove the
deal does not satsify complete fairness test.
2. A recent Delaware ruling found that if a Δ satisfies hatches 1 AND 2,
then business judgment rule protection returns.
v. In Texas, using any of the three hatches makes the transaction legal.
b. Stewart Interested Director Standard
i. A director is not independent, and is therefore interested, if there is a potential
personal benefit or detriment to the director as a result of the decision. In that
case, the director’s business judgment might be influenced by the personal
consequences of the decision.
1. In this case, Stewart is interested based on the potential criminal
implications of the illegal stock transactions.
2. Patrick is also interested based on her position with MSO and
compensation received from MSO.
3. However, by itself, Stewart’s friendship with Martinez and Moore does
not make those directors interested. There must be additional
circumstances indicating that the directors would put their friendship
ahead of their reputations.
4. Finally, Seligman’s call to the publishing house does not create
reasonable doubt about her independence. That call protected MSO and
its reputation in addition to Stewart’s personal reputation.
c. Cookies Food Products v. Lakes Warehouse Distributing, Inc.
i. Herrig uses his auto parts distribution network to expand the market for Cookies
Barbecue Sauce.

50
ii. Cookies shareholders sued Herrig based on four actions:
1. The distribution agreement using Herrig’s company.
2. The royalty agreement for the taco sauce Herrig invented.
3. The consulting agreement paid to Herrig.
4. The storage agreement using another of Herrig’s companies.
iii. All of the above qualify as self-dealing under DGCL 144.
iv. You could argue that no one on Cookie’s board is disinterested because Herrig
chose them all.
1. You could also argue that four of them are disinterested. Under Delaware
law, you have to prove the directors are in the self-dealing director’s
pocket somehow if they’re not self-dealing themselves (the “Martha
Stewart Rule.”).
v. Herrig is a majority shareholder. The statute doesn’t say that a majority of
disinterested shareholders has to approve.
1. However, it would be an absurd result if Herrig could bless the deal all
by himself.
vi. The Entire Fairness Test (apply instead of the BJR when there’s self-dealing and
no escape hatch is met):
1. Fair price:
a. Herrig’s contribution must be valued in comparison with the
amounts he paid himself.
b. The court says it should be on a value basis, which evaluates his
pay based on the value he contributed rather than what it would
cost to hire someone else. (Most courts use the cost approach,
though).
2. Fair dealing:
a. The self-dealing director has the burden to show the proper
procedures were followed.
d. Marciano v. Nakash
i. Nakash loaned money to Gasoline, Inc. The Marcianos complain that this was
voidable self-dealing.
ii. This type of transaction is illegal unless one of the three DGCL 144 escape
hatches is met.
1. Hatch One: In this case, there weren’t sufficient votes of disinterested
directors to ratify the transaction.
2. Hatch Two: The Nakashes were 50% owners and the Marcianos were
50% owners, so there wasn’t a majority of disinterested shareholders to
approve.
3. Hatch Three: Even if the transaction was fair, the Board must still
approve of it. The Board didn’t approve in this case, so this hatch also
isn’t available.
iii. The court finds that because there’s a deadlock on the board, the transaction isn’t
voidable per se as self-dealing.
1. The transaction was obviously fair and helpful to the company (the
interest rate on the loan was 1 point above prime, which only the
healthiest companies get).

51
iv. The Marcianos complain that the court is doing an end run around the parties’
business arrangement. They were deprived of their veto power.
v. With 1) deadlock, and 2) because of the abundant evidence of the deal’s fairness,
the court will approve the deal even without the Board’s approval – disregarding
the statute as well as the bylaws and certificate.
vi. Also seems to violate agency law: how can Marciano bind Corp to borrowing
without board approval?
1. Never assume that clearly written statutory language will carry the day in
the face of contrary policy interests.

CC. Corporate Opportunity Doctrine


a. All forms of business organization have a doctrine which forbids co-adventurers from
stealing business from one another.
b. People with a duty of loyalty to a corporation can’t take corporate opportunities for
themselves.

c. Northeast Harbor Golf Club v. Harris


i. Harris bought two parcels of land and developed them for housing.
ii. The first question in a Corporate Opportunity situation is to determine whether it
was actually a corporate opportunity.
1. The Line of Business Test
a. Is the corporation financially able to pursue the opportunity?;
b. Is it in the same line of business?; AND
c. Is it of practical advantage to the corporation?
2. Interest or Expectancy Test (TX)
a. Has the corporation chosen to make the opportunity of interest to
the corporation?
i. This is both narrower and broader than the Line of
Business Test.
ii. The corporation may be looking to go into new lines of
business (broader than LOB) or it may not want to
expand the current line (narrower).
3. ALI Test
a. How did the opportunity come to the corporation’s attention?
i. Context (was the information given to an individual or to
the corporation?)
ii. Did the director use the corporation’s resources to
discover the opportunity?
iii. The court applies the ALI test.
1. Harris learned of the opportunity while talking to the postmaster and
when a real estate broker called her up.
a. The broker thought he was dealing with the corporation,
therefore it was the corporation’s, not Harris’s.
2. The postmaster opportunity is trickier, because the information was
exchanged on a golf course. Could go either way, because it was
informal, but it was on the corporation’s property.

52
d. Broz factors for determining what is a corporate opportunity (Delaware):
i. Affirmative factors:
1. Corporation was financially able to pursue the opportunity?
2. Within the corporation’s line of business?
3. Corporation had an interest or expectancy in the opportunity?
4. By stealing the opportunity, the fiduciary will be put in a position
inimical to her duties to the corporation.
ii. Negative factors:
1. Opportunity is presented to the fiduciary in her individual capacity.
2. Opportunity isn’t essential to the corporation.
3. Corporation has no interest or expectancy in the opportunity.
4. Fiduciary hasn’t wrongfully used corporate resources to pursue the
opportunity.
e. Broz is an amalgam of other tests. It’s a big balancing test. Really there are only 7 factors
because one is duplicative.
f. Ostrowski v. Avery (Connecticut)
i. Avery tries to start his own cutting wheel manufacturing company, angering his
corporation. They also make cutting wheels.
1. Apply the LOB test:
a. On the one hand, the Corporation also makes cutting wheels.
b. On the other, they don’t make big cutting wheels.
2. I&E Test:
a. The corp. decided not to make big wheels, but then left the
possibility open, so this test isn’t clear, either.
3. ALI Test:
a. The big cutting wheel business is closely related to the small
cutting wheels business.
4. The Broz Test (Delaware):
a. The big cutting wheel factory may be in competition with the
small cutting wheel factory.
5. The Avowed Business Purpose Test (Connecticut)
a. Disclosure is an absolute defense. (same requirements as DGCL
144)
b. Without disclosure, fiduciary can offer clear and convincing
evidence that taking the opportunity didn’t harm the company.
i. Trier of fact gives special weight to any harm resulting
from nondisclosure.
g. In ALI jurisdictions, there’s no defense based on the corporation’s lack of financial
capacity.
i. However, disclosure (same as DGCL 144) is a “safe harbor” for taking a
corporate opportunity.
h. Even if you’re allowed to pursue an opportunity, you still have obligations as a fiduciary.
i. You can’t compete harmfully with your corporation.
ii. Can’t use corporate resources on your separate venture.
iii. Can’t disclaim your duty of care (e.g. – can’t start working 5 hours a week).

DD. Executive Compensation (A Form of Self-Dealing)

53
a. This is the most important and common self-dealing scenario.
i. Unavoidable
ii. Biggest opp. For stealing
iii. Biggest agency costs (big corp)
iv. Big minority oppression risk (closely held corps)
b. Executive compensation decisions are protected by the business judgment rule as long as
the DGCL 144 requirements are met (i.e., disclosure and approval by a majority of
disinterested directors and shareholders).

c. Wilderman v. Wilderman (Delaware)


i. Issue: Is an executive paying himself a self-dealing transaction covered by
DGCL 144?
1. Though the Delaware courts never cite that statute, they apply an
identical standard. Also, DGCL 144 should apply by its terms.
ii. Wilderman doesn’t fit in any of the DGCL 144 escape hatches.
1. Instead, as in Marciano, the court says the salary is OK as long as
Wilderman can prove it’s fair.
iii. Another issue: Is the company paying de facto dividends? (The director has the
burden of proving that it was a reasonable salary, not a dividend).
1. The IRS certainly thinks so. The IRS only allows a “reasonable salary” –
everything else is a dividend and is taxed as such (twice).
2. These are badly disguised de facto dividends because the salary rose
disproportionately to the company’s profits.
3. The court relies on the IRS’s determination and finds that the reasonable
salary is $45,000 – everything else is dividends.
a. Fairness disguised as quantum meruit
iv. The court finds that the salary was not fair within the meaning of 144.
1. Π’s veto power was overridden. (unfair dealing)
2. Π’s salary didn’t increase along with Δ’s, which is unfair. (unfair price)
d. In a closely held corporation, unfair salary problems involve shareholder oppression.
e. In a publicly traded company, the issue is one of agency costs – the directors are getting
paid exorbitantly at the expense of shareholders.
f. How to get biz judg so you can pay executives however much you want:
i. Hire compensation consultant
ii. Have the salary committee filled with disinterested directors and get it approved
by a board consisting of disinterested people. Under these circumstances, the
BJR applies.
1. Everyone on these boards knows the drill, and they rubber stamp each
others’ pay packages.
iii. This is easy to do, so there’s basically no viable legal challenge to executive pay.
The legal system creates an agency cost problem.
iv. See fed req. of salary disclosure for public corps (precatory)

EE. Majority Shareholders’ Duties


a. Sinclair Oil Corp. v. Levien (Delaware)
i. Sinclair Oil is a controlling shareholder of SinVen, a Venezuelan subsidiary.

54
ii. Πs sue Sinclair on claims that:
1. Sinclair took SinVen’s corporate opportunity.
2. Sinclair made SinVen pay too many dividends.
3. Sinclair breached a contract by failing to make minimum purchases of oil
and refined products and to pay upon receipt of goods.
iii. 1) Differential Impact?
1. if there is no differential impact in terms of costs and benefits, the BJR
applies
a. Differential impact  Entire Fairness Test
2. Courts won’t look behind the majority SHs’ purpose in giving out
dividends because there is no differential impact.
iv. The dividend transaction gets BJR review because both majority and minority
shareholders get the benefit of the dividends.
v. As for the corporate opportunities taken, it’s not a close question that the oil
opportunities taken by Sinclair were not in the line of business of SinVen, so no
problem there. BJR review.
vi. The breaches of contract were definitely self-dealing,
vii. 2) Rational business reason?
1. When the cost of capital is lower than the returns of the opportunities
available to it, a corporation should reinvest profits. When the cost is
higher than returns, a corporation should pay dividends.
2. Π argues that Sinclair was only trying to help itself by paying out the
dividends, so there was a differential impact.
3. Holding 2: The court will not look behind the shares to evaluate the
board action’s impact on the shareholders.
viii. Questions

FF.Sale of Control
a. Basic Rule: Control is a property right of the majority shareholder, who can sell it as she
pleases.
i. Exceptions:
1. A majority SH has a duty not to sell its controlling interest if it knows
looting will result.
2. Also can’t sell assets for personal purposes.
3. Can’t resign the Board after selling less than a controlling share.

b. Zetlin v. Hanson Holdings, Inc.


i. Market price of the stock is $7.38. The buyer paid $15, a more than 100%
premium for control.
ii. Π, a minority SH, sued for a cut of this premium.
iii. Share is only controlling if large enough that it can elect majority of board
iv. Controlling Share companies are more valuable than ones with lots of minority
shareholders b/c agency costs reduced
1. controlling SH has strong incentive and more ability to monitor
management closely
2. No free rider problem (fire useless employees unilaterally)

55
3. Controlling SH decisions protected by biz judge rule
v. Reasons for Controlling Share Premium (~100%)
1. Synergy
2. Improved management
3. Control
4. The stock is undervalued.
5. Maybe you want opportunity for Self-Dealing/Looting.
vi. 1-2 are socially positive, 3-4 are neutral, and 5 is socially negative.
vii. Controlling SH hurts minority
1. Discount to min. share value (~35%)
2. Oppression opportunities (Looting, freezouts, compensation)
3. Stranger danger
viii. Lenient general rule means society values chances of good reasons more than
risk of good ones
c. Equal Opportunity Rules
i. Why isn’t it better to pool all the stock together and have the buyer purchase a
pro rata controlling stake from the pot rather than purchasing it only from the
majority shareholder?
1. Neither the current controller nor the prospective controller would be on
board with that So a lot of sellers wouldn’t sell and vice versa
a. Seller won’t like it because they’d have to become minority
shareholder, split the premium, cant get out of company .
b. Buyer won’t like it because largest minority shareholder will
have been a majority shareholder previously
2. Hurts value of all corporations, less reason for premium
ii. Why not the 100% rule?
1. Many buyers cant afford entire company,
2. Forces everyone to sell their stake regardless of what they want
d. Gerdes v. Reynolds
i. Δs sold their controlling stake, resigned immediately, then allowed the new
shareholders to loot the company.
ii. It’s illegal to turn over the Board if you sell less than a controlling stake of the
corporation’s stock – otherwise it’s fine.
iii. The company’s assets were worth $0.06 per share. The buyers paid $2 per share.
The court says they’d never do that unless they were going to loot the company.
iv. Securities company assets are liquid: easy to loot
1. Even though Δs did an investigation on the buyers, and got the false
impression that they were friends with the Rockefellers.
v. Majority: Red Flag Rule: If there’s enough indicia that the buyers are looters,
then the sellers have a duty to do an adequate investigation.
1. On notice if poor investigation & strong indicia
2. Contra caveat emptor
vi. Minority: proper investigation always required
vii. Remedy:
1. Difference between purchase price and reasonable purchase price
a. The Court ordered the sellers to give back the “blood money” –
the amount of the premium that was paid for the looting, but

56
allowed that some of the premium was for legal premium goals
(synergy, undervaluation, etc.).
2. Disgorgement:
a. Even if there is no stealing, still have to pay back the illegal
premium.
3. Huge boondoggle for SHs
viii. So, always do an investigation of the purchasers.
1. The strength of these protections tilts in favor of protecting minority SHs
rather than encouraging value-creating transactions.
a. Clear likelihood of looting impairs value add policy, helps
minority protection policy
e. “Other Exceptions?”

GG. Indemnification
a. § 145 gives three flavors
b. Directors can’t be sued for damages for breach of duty of care.
i. However, the duty of loyalty can’t be disclaimed completely
c. In Delaware, to be eligible for indemnification, you must be sued
i. 1) in your capacity as a director/officer or
ii. 2) by reason of the fact that you were a director/officer.
d. mandatory indemnification
i. when you’re successful in your defense whether or not it’s on the merits.
e. permissive indemnification,
i. Who can do it?
1. A majority of disinterested directors
2. A committee of disinterested directors
3. A majority of shareholders
4. Independent legal counsel in a written opinion (if there are no
disinterested directors, or if the disinterested directors order it).
ii. Req.s for Director?
1. acted in good faith and with reasonable belief that you’re acting in or not
opposed to the corporation’s interest
f. Advancing indemnification is OK, but it should be in writing because the director may
have to give it back.

g. Waltuch v. Conticommodity Services


i. Waltuch is sued for fraudulent silver transactions. They were market
manipulations, fraud, and antitrust violations.
1. All the claims in civil court were settled out of court.
ii. Was the insider trading done by reason of the fact he was a director?

57
1. He was trading from his personal account not corp assets– the fraud was
based on silence regarding his insider knowledge. (Silence is only a fraud
when you have a duty to disclose).
2. Some courts hold that when what you did is only illegal because you’re
an officer or director, it counts as being sued by reason of the fact you’re
an officer or director.
iii. Must the corporation indemnify Waltuch?
1. He was successful in the civil suits because they were settled and the
company paid for the settlement. Waltuch incurred no personal liability,
therefore he was successful.
iv. Merrit-Chapman says you can get partial indemnification if you win on some
claims but not others.
v. Is Waltuch entitled to permissive indemnification?
1. No, because you have to have (DGCL 145(a)).
vi. Waltuch argues that despite the 145(a) requirement of good faith, Article 9 of the
corporation’s Articles of Incorporation says the corporation “shall indemnify,”
with no good faith requirement.
vii. DGCL 145(f) says a corporation can indemnify beyond the statute, so Waltuch
says Article 9 is legal in its indemnification despite bad faith.
viii. However, Waltuch’s interpretation would make 145(a) meaningless.
ix. Holding: The court holds that 145(f) can grant broader indemnification, but it
must be permissible under 145(a).
1. E.g. – A corporation can advance money, which isn’t explicitly allowed
in (a), but isn’t disallowed, either.
x. Also, 145(g) is an exception to 145(a) because it allows D&O insurance to
indemnify.
1. It too would have no meaning if Waltuch’s 145(f) argument were correct.

HH. Duty that Shareholders Owe Each Other


a. Minority shareholder’s plight: Minority shareholders in a closely held corporation are
especially vulnerable. There’s no market for their shares, so they have no way of getting
money out of their investment if the majority shareholders freeze them out.
b. Majority shareholders can freeze out minority shareholders if the law didn’t step in. This
is done by taking away all the minority shareholders’ ability to make money from the
corporation, then force them to take a bad deal to sell their shares back to the majority.
i. Classic Freeze-Out Scenario:
1. Self-deal (steal revenue from the minority by taking corporate
opportunities for themselves, etc.)
2. Withhold dividends
3. Stop sharing info
4. Fire them from jobs
5. Remove them from Bd
ii. How to cover this up and hide it from the shareholders?
1. Keep them off of the Board so they have no right to demand the books
and information.
iii. Then, you offer them a steep discount to buy out the minority.

58
c. Donahue v. Rodd Electrotype Co. (Massachusetts)
i. In a closely held corporation,
ii. The Donahues are mad because the Rodds bought shares from their family
members at $800 a share without offering a similar deal to the Donahues.
1. What evidence that this was inflated?
a. They offered to buy Donahues’ shares at a maximum price of
$200.
iii. In this scenario, the entire fairness test applies (because there’s self-dealing & no
exception).
iv. Holding: The court holds that shareholders in a closely held corporation owe one
another the fiduciary duty that partners owe one another (“a punctilio of honor
the most sensitive…”):
1. they expect to manage the company going in, make a living this way, not
just passive SHs
a. If you’re not employed by the company and you’re not getting
dividends, you have no way to get money from the company.
2. Social benefit: Increases min SH protection
3. Social cost: Decreases flexibility of majority shareholders
a. Invites frivolous lawsuits: oppression by minority
4. It makes sense only if the danger of shareholder oppression is greater
than the cost of limited flexibility. It is a drag on the market.
a. However, Ragazzo thinks it’s warranted.
5. Really just a duty of fairness
a. See Meinhard: partners can still go after profits, vote shares
selfishly
v. Holding 2: The Equal Opportunity Rule [CA]
1. Benefit maj gives itself must be offered to min
2. Rodds must offer to buy all of the Donahues’ shares at the same price. It
could be argued this is better than an equal opportunity, but on the other
hand, it’s the opportunity that matters, not the dollar amount).
a. Similarly situated people should be treated similarly & vice
versa, says Aristotle.
b. Donahue, then, must be treated dissimilarly from Rodd.
3. The Equal Opportunity Rule is almost impossible to apply. It requires a
value judgment as to what counts as similarly or dissimilarly situated.
Only California applies it.

d. Wilkes v. Springside Nursing Home, Inc. (Massachusetts)


i. Wilkes was frozen out of the company:
1. Not elected to office/board.
2. Fired from employment.
3. No dividends.
4. Offered a deflated price for his shares.
ii. The Donahue standard would require an equal opportunity given to Wilkes.
1. The other shareholders all were on the Board and all had jobs and
salaries while Wilkes did not.

59
iii. Holding: Legitimate Business Purpose Test. The Wilkes court decided Donahue
was too broad and difficult to apply. So, instead, they apply a balancing test
which weighs the legitimate business purpose and the practicality of a less
harmful alternative.
1. Courts generally require the alternative to be as efficient as the disputed
action.
iv. The rules are different in a publicly traded company – you can’t sue one of those
for firing you, withholding dividends (see Sinclair Oil), etc.
1. Every element of the freeze-out scheme would be legal in a publicly
traded company: passes BJR but not EFT
2. Wilkes accepts the premise of Donahue, that minority shareholders in
closely held corporations get special treatment.
v. Relief
1. Compensation for past bad acts, but not future ones
2. Corp. does not want to supervise acrimonious relationship
vi. If maj was just streamlining and fired everyone, stopped all dividends, no rule
would help min SH
1. Thus, if clear legit biz purp  no less harmful alternative
2. Even if wilkes only invested to get a job and relied on promise that he
could keep it, he still loses though it is unfair b/c no less harmful opp

e. Merola v. Exergen Corp. (Massachusetts)


i. In this case, π loses because the company was paying dividends and he was an at-
will employee. He had no investment to protect.
ii. The Court also makes a big deal out of the fact Merola wasn’t a founding
shareholder. However, this fact probably doesn’t matter.
iii. What is important is that Merola was paid a fair price for his stock.
1. There wasn’t a connection between his employment and his investment –
he wasn’t depending on his salary to get a return on his investment.
f. Smith v. Atlantic Properties (Massachusetts)
i. The IRS assesses penalties on the company for withholding dividends unduly.
They penalize for the years 1962–68.
ii. There’s a provision in the Articles that requires an 80% shareholder vote to
approve any board action.
1. This is illegal under Long Park because it would sterilize the board.
2. How to make this legal?
a. Become a statutory close corporation under DGCL § 342- §
343.
b. § 350 allows restriction of the board of close corporation by
holding a majority of outstanding stockholders to vote
3. At any rate, the court missed the illegality of the provision.
iii. Wolfson didn’t want to pay dividends and used his veto to prevent paying them
out.

60
iv. There is a legitimate business purpose for withholding dividends. Why doesn’t
the business judgment rule protect the decision to withhold dividends in this case
(as was the case in Sinclair Oil)?
1. The court says Dr. Wolfson’s acting in his own self-interest. It’s self-
dealing because Dr. W is in a higher tax bracket than the other
shareholders, so he benefits more from withholding dividends.
2. He also doesn’t give a good reason for withholding dividends
3. In Delaware, this would be perfectly legal because, per Sinclair Oil,
courts can’t look into the motivations for or indirect effects of a
transaction that has no differential impact on different SHs.
v. In this jurisdiction, the self-interest not shared by the other shareholders is
enough to find self-dealing.
vi. Also, Dr. Wolfson didn’t have any plan in place for what to do with the money
saved by not giving out dividends.
1. So, he breached his duty of care.
2. Veto power subjects him to duty of care, normally reserved for Directors
vii. It doesn’t matter that he took a risk which materialized. What matters is that there
was self-dealing and a breach of the duty of care.

II. Shareholder Oppression: Dissolutions and Buy-Outs


a. Shareholder Oppression Test: Look to the reasonable expectation of the shareholder at
the time the investment was made.
b. Non-transgressable interests
i. Employment
ii. Dividends
iii. Participation
iv. Protection against self-dealing
v. Profection against forced-sale

c. In Re Kemp & Beatley, Inc. (New York)


i. The company never paid out actual dividends, but did pay out de facto dividends
in the form of bonuses paid out in proportion to the amount of stock the
shareholders owned.
1. This was because the company gets a deduction for paying salary, but
not for paying dividends.
2. Everyone was happy with this scheme until Gardstein and Dissen were
fired and no longer got the bonus/de facto dividend.

61
ii. In Massachusetts, they would have applied the legitimate business purpose test,
which the πs would have won.
iii. Under New York law, a shareholder can petition the court to dissolve the
corporation if there’s shareholder oppression.
iv. Reasonable Expectation Test: Gardstein and DIssen expected to have a job
because they worked there for years and years – as long as they had shares they
had a job.
1. This is a specific expectation depending on the circumstances.
2. How to determine what they expected?
a. Look at the course of business and facts.
v. The Court found oppression and ordered dissolution unless the majority bought
out Gardstein and Dissen at fair market value.
vi. Difference between the MA and NY tests:
1. In MA, the majority’s purpose is dispositive.
2. In NY, the minority’s reasonable expectation is dispositive.
vii. In Wilkes, if the majority had decided they needed better management and
decided to fire Wilkes, he might have lost in MA, but not in NY.
1. Most of the time, the NY and MA tests are the same, but not always.

d. Davis v. Sheerin (Texas)


i. The jury found the majority shareholders…
1. Conspired to deprive minority shareholders of stock ownership
2. Paid informal dividends into a profit-sharing/pension plan.
ii. Problems with the Wilkes remedy:
1. Fixing the current oppression won’t solve future oppression.
2. Leaving Sheerin in the company is risky because of the acrimonious
relationship between the shareholders.
iii. Because of the above, the Court prefers the dissolution framework. 39 states use
this method.
iv. In Texas, there are intermediate steps before dissolution can occur. Must have a
receiver take over the company for a year.
v. To avoid this headache, the court orders the buy-out remedy.
1. Everybody gets what they want.
2. Reminiscent of P’ship law, except you have to show oppression
vi. The Texas statute doesn’t specifically allow the buyout remedy.
1. Dissolution is an equitable remedy. You are never entitled to an equitable
remedy.
2. The Court can’t grant dissolution unless the π proves oppression, but
even if oppression is proven, the court has discretion whether to grant
dissolution or not.
3. Courts in equity always have discretion to award lesser types of relief.
vii. A 2014 Texas Supreme Court case (Richie v. Ruth) changed the test for
oppression.
1. No oppression unless the majority hurts the company and an identifiable
group of investors.
a. This is a very high bar. Most of the classic freeze-out factors
(except improper self-dealing) wouldn’t qualify.

62
2. Also found that the statutory remedy of dissolution is exclusive. No buy-
out remedy is allowed.
3. This makes Texas the least protective jurisdiction for the rights of
minority shareholders.
viii. Davis is still emblematic of the law around the country.

e. How to fix the buyout price?


i. If minority discounts are applied to the buyout price, it is for reasons of:
1. Lack of control
2. Lack of liquidity/marketability
a. Usually there are restrictions on transferability and there is no
market for the shares.

63
ii. Some would argue that a buyout shouldn’t put the minority shareholder in a
better position than if he tried to sell on his own.
1. Counter-argument: Having to sell at all is a harm when there’s no market
for the shares. The undiscounted buyout protects the minority
shareholder from that harm.
iii. The majority rule is that there should be no minority discounts applied to a
buyout price.
iv. A substantial minority of jurisdictions will apply a liquidity discount.
v. Another problem: When should you value the company?
1. If you use the date of filing of the complaint, the majority may have
already run the company into the ground.
2. The majority rule is to use the date of filing with equitable adjustments.

JJ. “No Special Rules” Jurisdictions (Contracts Trump Minority Shareholder


Protection)
a. Nixon v. Blackwell (Delaware)
i. The πs complained that the majority set up an ESOP and a key-man insurance
program that favored shareholder employees over regular shareholders.
ii. The corporation provided a measure of liquidity to the employee stockholders
through two mechanisms. Upon retirement or termination of employment,
employees were entitled to receive their interest in the ESOP in Class B shares or
cash in lieu of shares. Upon death or termination of employment, each Class A
shareholder gave the corporation the option to call his Class A shares and
substitute Class B shares. The corporation purchased key man life insurance
policies so that it would have the funds to purchase Class B shares from the
estates of deceased executives.
1. This creates a guaranteed market for the employees’ stock and prevents
the stock’s transfer to the decedent employees’ heirs.
iii. How other j(x)s would handle it:
1. California: The πs would definitely win under Donahue because the
majority shareholders, who are ESOP holders, get to sell back to the
company while the class B stock holders can’t.
2. Massachusetts: Under the Wilkes legitimate business purpose test, the πs
would probably lose because the incentivizing mechanism of the ESOP
is a legitimate business plan.
a. The company doesn’t buy back Class B stock because it’s non-
voting, and buying back Class A stock lets the current
management remain in control of the company – also a
legitimate business purpose.
b. Is there a practical, less harmful alternative?
i. No, because buying out the Class B shareholders would
cost a lot more money. It’s not as efficient as the current
plan, so the second half of Wilkes tilts in favor of the Δs.
3. New York: Kemp Test (reasonable shareholder expectations):

64
a. Hard to apply in this case because the minority didn’t make any
investments.
b. IN this case, you’d look to the shareholders’ expectations when
they received the stock from Mr. Barton (the company’s
founder).
i. The expectation in this case comes from Mr. Barton’s
plan, so πs won’t win under Kemp.
iv. Instead of the special rules used by other jurisdictions, the Entire Fairness Test
will apply because of the self-dealing and differential treatment (the majority
shareholders were bestowing a benefit on themselves and not the minority).
1. Delaware takes the view that the statutory close corporation self-help
protections are sufficient to protect minority shareholders in a closely
held corporation. In a statutory close corporation, shareholders can
bargain for:
a. Veto power (sterilizing the board)
b. Dissolution at will.
v. Policy: Is Delaware’s view fair?
1. Yes:
a. You would have to pay more per share for the above protections
– you shouldn’t get them for free.
2. No:
a. Circumstances of future oppression are hard to foresee
b. Contracting/negotiations don’t really occur in most small
corporations, and they may be hard for a court to enforce as
discussed in Kemp.
c. Small corporations and their shareholders have much less access
to expensive legal counsel.
i. Smaller the enterprise, the larger the relative transaction
costs of having lawyers negotiate a robust contract
protecting both parties
b. The vast majority of other jurisdictions don’t take Delaware’s view.
c. Would Wilkes have prevailed in Delaware?
i. The business judgment rule would apply because there’s no differential impact
on the shares, therefore there’s no self-dealing (see Sinclair Oil).
1. The firing didn’t affect the shares.
2. The lack of dividends didn’t differentially impact his shares.
3. There is no review of choosing who’s on the board.
a. It’s not reviewed by any of the above rules.
d. Delaware courts have been coy about whether Nixon really means there are no special
rules for closely held corporations.
i. Three possibilities:
1. Nixon means what it says – freeze-outs are OK.
2. The courts will distinguish Nixon and measure differential impact
amongst shareholders to find self-dealing.
3. The courts might massage the entire fairness test to find that a freeze-out
will fail that test.

65
KK. Contract vs. Fiduciary Duty and Oppression
a. Gallagher v. Lambert (New York)
i. Gallagher signed an at-will employment contract with his corporation, which also
allowed the corporation to buy back his stock at book value when he left.
ii. After the stock price rose, the corporation fired him for the purpose of buying
back his stock at the lesser book value.
iii. Objective theory of contracts:
1. Gallagher probably would not have bought the company stock if there
were an understanding that he could be fired so the company could buy
back the stock at book value.
a. Had they discussed it, the parties probably wouldn’t have agreed
that the company can steal back the stock by firing Gallagher.
2. Reasonable people would likely find that there was an exception to the
at-will employment in cases where the employer fires the employee just
to steal the stock back.
3. Fiduciary duties begin where contracts end. They fill up the areas the
contract doesn’t cover.
a. There are plenty of situations where a contract is unambiguous in
not including restrictions, but there are implied universal
restrictions.
iv. Holding: The firing was perfectly legal under the contract. At-will means at-will.
Shareholder rights and employee rights are separate, not combined – the effect on
Gallagher as a shareholder doesn’t matter for the inquiry as to the
appropriateness of the firing.
1. Jurisdictions are split 50-50 on this view.

LL. Dissolution for Deadlock


a. Wollman v. Littman (New York)
i. The Nierenberg sisters allegedly set up a separate company and lured away
Chevreau’s clients – the Littmans sued for usurpation of corporate opportunities.
1. The Nierenbergs counter-sued for deadlock and asked for dissolution of
the corporation.
ii. Page v. Page said that setting up a company for the purpose of usurping
opportunities is legal until the opportunities are actually usurped.
iii. The Nierenbergs’ plan is to create deadlock so the corporation will be dissolved,
giving them free reign to take the clients.
iv. The court appointed a receiver to maintain the business while a finding of facts
takes place – if it’s true that the Nierenbergs intend to steal the company through
deadlock, then dissolution won’t be granted..
v. Dissolution by court order is hard to get.
vi. Del
1. No dissolution for oppression
2. No dissolution for deadlock unless there are only 2 SHs
3. Can appoint a custodian/receiver under § 226 to manage the business in
lieu of the board

66
MM. Securities Fraud
a. General partnership interests are typically not securities.
b. Stock in a corporation is always a security.
c. Federal securities law is generally more pro-π than the common law of fraud.
i. It’s a good idea to bring common law fraud and federal securities claims
together, though, because common law fraud allows punitive damages, while
federal securities law has the exceptions to the reliance requirement.
d. Elements of a federal securities claim:
i. A lie.
1. Can be a misrepresentation, half-truth, or silence when there’s a duty to
disclose.
ii. Materiality of the lie.
1. A reasonable person would want to know it when making an investment
decision.
2. In the case of silence, it’s only a lie if the undisclosed information is
material.
iii. Scienter
1. In the case of an affirmative misrepresentation, the Δ must knowingly or
recklessly lie.
2. For an omission, Δ must know or be reckless to the fact that the omission
is misleading.
iv. Reliance
1. The victim/π actually made a decision based on the lie.
2. EXCEPTIONS:
a. When there is an omission, reliance is presumed. Therefore, π
does not have to prove he wouldn’t have done the transaction
had he known the bank hadn’t categorically declined the loan.
i. Policy: It’s hard to prove reliance in an omission case.
b. When there has been a fraud on the market, and an efficient
market has mispriced a security due to the Δ’s fraud.
v. Loss Causation
vi. Damages

e. Michaels v. Michaels
i. Lie 1: Δ told π that the bank said “no” to a loan request. This is a half-truth
because they would have given the loan if Δ had been willing to make a personal
guarantee.
1. If the bank didn’t say “no” categorically, it’s likely another bank would
have been willing to lend the money.
ii. Scienter: Scienter is tough in this case because, from Δ’s perspective, the bank
had turned them down.
iii. Reliance: Also a tough issue because the information was slight.
iv. Lie 2: They didn’t tell π about the meeting with Goldman Sachs.
1. Δs owed a fiduciary duty to π, a minority shareholder, per Sinclair Oil.
v. Was the meeting material? Look to how the Δs treated the information.
1. In this case, Δs got everyone together to “get ducks in a row” and said,
“Nobody tell [π]!”

67
vi. Lie 3: Δs didn’t tell π about the buyer.

NN. Certificate Amendments


a. Certificates can be amended subject to two broad requirements:
i. An amended certificate may contain only such provisions that could be lawfully
put in the original certificate.
ii. If a change in the rights of shares or shareholders, or an exchange,
reclassification, or cancellation of shares or rights is to be made, the provisions
necessary to effect such change, exchange, reclassification, or cancellation must
be set forth in the amendment.
b. Who can amend?
i. If no money has been received from shareholders yet, a majority of directors can
amend the certificate (or the incorporators if no directors are named).
ii. If money has been received from shareholders, an amendment begins with a
formal resolution by the board setting forth the proposed amendment and
directing it to be submitted to a vote by the shareholders. Then, written notice
setting forth the text or a summary of the amendment must be given to each
shareholder (DGCL § 242; MBCA § 10.03).
1. In Delaware, a majority of all shareholders entitled to vote must vote for
the amendment (DGCL 242(b)(1)). For ordinary matters, shareholder
approval is granted with a majority of shareholders present at the
meeting.
2. MBCA jurisdictions require a majority of shareholders entitled to vote as
a quorum requirement for the meeting. Then, a majority of the quorum
can approve the amendment.

c. Shanken v. Lee Wolfman, Inc.


i. Article 4.02 of the T.B.C.A. says that if you try to amend the certificate so as to
change the number of shares of a class, then a 2/3 vote of each class of shares is
required.
ii. In this case, Class A and B didn’t get 2/3 of C on board, so they went to Plan B,
which increased the number of authorized, but unissued shares in Class A and B.
1. This allowed Class A and B to collectively vote with 2/3 of the total
shares and exclude Class C from a class vote.
iii. However, for any amendment that changes the relative rights of the shares, there
must be a class vote.
1. It seems like increasing the authorized shares dilutes Class C’s voting
power. Yet, the court – and courts across the land – holds that the change
does not affect relative rights.
a. The court won’t look past the effect on the shares to the effect on
the shareholders.
b. The rights of Class C stock has not changed, it’s just that the
shareholder will own a smaller share of the stock.
iv. Rule: Merely issuing more shares does not trigger a class vote.
1. A class vote is required when the rights of a class are changed, or when
the number of class shares is changed.

68
v. The board can’t issue more shares than the certificate authorizes.

OO. Bylaw Amendments


a. In Delaware, the bylaws are adopted by the directors at the first board meeting.
Subsequent amendments are voted on by the shareholders entitled to vote.
i. DGCL 109: The certificate may give directors the ability to amend the bylaws,
but the shareholders retain the power to adopt, amend, or repeal bylaws even if
the board has been given such power.
b. In MBCA jurisdictions, the directors have the ability to amend bylaws unless (1) the
certificate says they can’t or (2) the shareholders adopt a bylaw that says directors can’t
amend bylaws.

c. Keating v. K-C-K Corp. (Texas)


i. The bylaws of the corporation require the election of 3 directors. However, after
several years of electing 3 directors, the shareholders elected 4.
ii. The corporate charter required a 2/3 shareholder vote at a shareholder meeting to
amend the bylaws.
iii. Holding: The court finds that longstanding practice or custom can amend the
bylaws of a closely held corporation. (Delaware has the same rule announced in
In re Osteopathic Hospital Association).
1. Contrast with Nixon v. Blackwell, which held there are no special
exceptions for closely-held corporations.
2. Public companies have a different rule because shareholders expect all
the formalities to be followed.

PP.Sale of Assets and Appraisal


a. Texas rule: There must be board approval and a 2/3 shareholder vote to sell corporate
assets unless it’s in the regular course of business.
b. DGCL 271: A majority of outstanding shares entitled to vote must approve before
“substantially all” corporate assets can be sold.
c. Shareholders are allowed to vote on those decisions and on changes in the certificate
because such decisions change the nature of their investment.
i. No right to vote means no right to appraisal
d. Dissenters to § 262 transactions have a right of appraisal / right of redemption, which
must be perfected. This is allowed because the nature of the investment has changed.
i. Perfection requirements:
1. The corporation must notify the shareholders within 20 days of the vote
that appraisal rights are available.
2. The shareholder must make a written demand for appraisal prior to the
vote and must not vote in favor of the sale.
3. There is a statute of limitations of 120 days of the effective date of the
transaction to petition the court for an appraisal.
ii. If all of the above are met, then the shareholder is awarded a “fair” value for the
stock – no minority discount, but also no value added from the transaction you
voted against.

69
e. Del. doesn’t grant appraisal for sales of “substantially all” assets: has to be in the form of
a merger
f. No appraisal or approval of assets sold in the regular CoB,

g. Rudisill v. Arnold White & Durkee, P.C.


i. AWD sells “substantially all” of its assets – it retained vacation homes, insurance
policies, and cars.
ii. The court held that the sale with the other firm was in the ordinary course of
business.
1. This comes from the TBCA, which defines “regular course of business”
broadly – any sale of less than all assets with consideration counts.
iii. The legislature was reacting to the Gimbel-Katz framework, where a sale of
assets “quantitatively and qualitatively vital to the operartions of the corporation”
is a sale of “substantially all” assets.
iv. So, the Texas legislature registered its disagreement by enacting the TBCA
provision which broadened the “regular course of business definition.” This was
an overreaction.
1. The court reads the statute literally, such that any sale of assets for
consideration is in the “regular course of business.”
2. Courts can always choose whether to read a statute literally or by the
spirit of the law.
v. Hollinger distinguished from Gimbel-Katz by saying the viability of the business
must be in severe doubt after the sale of assets for it to be a sale of “all or
substantially all” assets.

h. Appraisal
i. Piemonte v. New Boston Garden Corp.
1. Every state in the country will give an appraisal for a merger.
2. The court applies the Delaware block method to evaluate the price of a
corporation that owns the Boston Bruins.
3. The DE block method takes into account:
a. Market value (FMV in private corp, last sale for close)
i. Liquidity and market discounts in last sale mean we
would weight this low
b. Earnings value
i. Our avg. E/S ratio (minus one off events) divided by
market value over last few years vs same for industry)
c. Net Asset value ($103.16) (if sold on mkt now)
d. Criticized b/c it leaves highest and lowest values on the bd.,
some types of company are clearly FMV (IBM); Assets (Energy)
4. The court applies different weights to each factor.
5. The safer the business, the higher the multiplier for earnings value.
a. Public co.s: avg. industry earnings per share divided by mkt
b. They don’t compare to other NHL teams in this case because the
Boston Bruins are in a different market and the team is really
good.

70
6. So, the court assigns an earnings value of $52.60 ($5.26 earnings
multiplied by 10).
a. It’s a quick and dirty method of valuation, and it’s less
susceptible to manipulation, objective – it’s the preferred method
for these reasons.
7. Court gives a 10% weight to market value, 40% to earnings value, and
50% to net asset value.
8. Discounted cash flow is the preferred method of appraising because it
predicts future earnings. However, in Weinberger, the Delaware supreme
court said that judges may use any appraisal method used by business
professionals.

i. Freezeout Transactions
i. Farnsworth v. Massey (Texas)
1. Majority shareholders sell the company’s assets to a new company which
they own, excluding plaintiff from participation in stock ownership in
new companies and causing his shares in old companies to become
worthless.
2. Texas Supreme Court: No recovery against the majority shareholders
because the proper procedures were followed.
3. The appellate court awarded $151,000, plus the award for “fraud,” which
was $54,000.
a. This is double-counting, because the $151,000 fully
compensated the minority (they don’t get the added value from
the transaction).
4. Holding: The court says appraisal is the exclusive remedy absent fraud
in the transaction (breach of fiduciary duty). Essentially, a freezeout is
OK if you pay fair value for the minority’s shares.
a. This is bad for minority shareholders when the transaction adds
value – appraisals never take into account value added by the
transaction.
5. Appraisal proceedings have some disadvantage vis-à-vis a breach of
fiduciary duty:
a. No class actions.
b. No attorneys’ fees.
6. The supreme court awarded the full $312,000 even though they ruled the
πs weren’t eligible for an appraisal.

QQ. Mergers
a. DGCL 251 governs mergers. The boards of both companies must approve the merger
agreement.
i. The majority of outstanding stock entitled to vote must vote in favor if the nature
of their investment is changing – e.g., if the company is being subsumed into
another company.

b. De Facto Mergers

71
i. Hariton v. Arco (Delaware)
1. Arco is selling itself to Loral in exchange for 283,000 shares of Loral
stock.
2. Why do the companies do this transaction as a sale rather than as a
merger?
a. The Arco shareholders won’t get appraisal rights under DGCL §
262, though they will get a vote.
b. The Loral SHs don’t get a vote or appraisal rights (no vote
because the nature of their investment isn’t changing, and there’s
no voting on the board’s business decisions).
3. In substance, the transaction is identical to a merger. However, in
Delaware, the courts will respect the form of a transaction.
a. The principle in Delaware is for each statute to have independent
legal significance. § 271 isn’t limited by § 251.
b. To read them otherwise would be in contravention of the
principle of reading a code as a whole.
4. Delaware’s approach is taken with business in mind – it ensures that
those transacting business can choose whatever form they want and
courts will respect it. It allows for efficiency and certainty.
5. This case is a hallmark of Delaware’s enabling philosophy toward
business law.
6. New York, on the other hand, recognizes the de facto merger doctrine. Π
would have won had they sued in New York.
7. Why have Arco sell to Loral rather than vice versa?
a. Loral was a New York company. They would have rights of
appraisal if they had sold.
8. Another benefit is that Arco will dissolve, and all liabilities of Arco will
be extinguished.
a. E.g. – If Arco made asbestos, victims won’t have anyone to sue
after this deal.
ii. Texas has statutorily rejected the de facto merger doctrine.
iii. Delaware’s doctrine of independent legal significance allows good lawyers to
make end runs around anything as long as you can find a statute that permits it.

c. Freezeout Mergers
i. Coggins v. New England Patriots (Massachusetts)
1. The Δ’s creditors wanted him to freeze out the other shareholders so that
he can put up the corporation’s assets as collateral for the loans.
2. To do this, Δ established a new corporation and merged the old
corporation into it, with the shareholders getting $15 per share.
a. § 251(b)(5) permits cash, property, rights, or securities as
consideration for the shares of the disappearing company.
3. The court says that merely following statutory procedures isn’t enough.
4. The test for whether a freezeout merger is appropriate is:
a. 1) Business Purpose Test (Wilkes); THEN
b. 2) Entire Fairness Test (Del.)

72
5. The NFL requires its teams not to be publicly traded so that no one
knows how much money they’re making.
6. The remedy can’t involve unwinding the merger – too much time has
passed and you can’t “unscramble the eggs.”
a. Instead, the court orders rescissory damages in the amount of the
current value of the stock.
b. This is more than just compensation because the original $15 per
share was a fair price.
c. Awarding damages beyond compensation is the only way to give
teeth to the idea that freezeout mergers are illegal.
ii. In Weinberger, the Delaware supreme court disclaimed the business purpose test.
Instead, they use the entire fairness test.
1. The burden would be on the π in Coggins because a majority of the
disinterested shareholders voted for the merger.
d. Dissolution
i. In Delaware, dissolution requires approval by the board and a majority of
shareholders entitled to vote (DGCL 275(a) and (b)). Under the Model Act, it
must be approved by the board and a majority of a quorum of shareholders (a
majority of shareholders entitled to vote is a quorum).

ii. Grato v. Grato (New Jersey)


1. You seldom get fair value when you’re getting dissolution proceeds.
2. The Grato court agrees with Coggins that a freezeout merger is illegal.
However, they have a different remedy:
a. The fair value of the stock prior to the transaction.
b. There’s not much difference between this and Farnsworth.
3. You can ask for an injunction before the merger’s execution in either
case, and you get an appraisal right (or the equivalent thereof) in both
cases.

RR. Derivative Suits


a. A derivative suit is a suit on behalf of the corporation against someone who has breached
a duty to the corporation.
b. Derivative vs. Direct:
i. A derivative claim is one that alleges a harm committed against the company.
ii. A direct claim is one in which the harm is against the specific plaintiff.
c. Why derivative rather than direct?
i. In a derivative suit, the corporation is an indispensable party – it is bound by the
judgment and can’t be sued again, therefore there’s less litigation.
ii. Creditors are protected.
iii. All the shareholders benefit because everyone’s a party.

d. Barth v. Barth
i. Π thinks Michael (Δ) has harmed him because Δ paid himself excessively
(derivative claim), used company assets for personal purposes (derivative), didn’t

73
give out enough dividends (direct), and appropriated corporate funds for personal
uses (derivative).
1. Massachusetts would say that all of the above are direct claims, while
Delaware would find at least the three of the four are derivative claims.
ii. Most jurisdictions require the π to have a special injury to lodge a direct suit.
1. In this case, you could argue that minority shareholders have a special
injury because the majority is stealing from the minority.
2. However, most states won’t find that minority shareholders have a
special injury because all of the shares are diminished equally. Under
Sinclair Oil, one looks to the effect on the shares, not the shareholders.
iii. Court Announces a special rule for closely held corporations:
1. RULE  In the case of closely held corporations, the court has
discretion as to whether or not to require shareholder to proceed under a
derivative action; determined by whether or not the policy reasons are
implicated in allowing a direct suit.
a. Test  Does allowing a direct action:
i. Unfairly expose the corporation or the defendants to a
multiplicity of actions?
RR> Policy of derivative actions is to put all the
claims into one suit.
SS> Materially prejudice the interests of creditors
of the corporation?
TT> Policy of derivative suits is to put money back
in treasury so as to keep it available to
creditors.
ii. Interfere with a fair distribution of the recovery among
all interested persons?
RR> Policy of derivative actions is to put it back
into the corporate treasury so that all wronged
shareholders benefit proportionately from the
recovery.
iv. Different j(x)s’ approaches:
1. DE: All are derivative, no distinction for closely held corporations
2. MA: The minority has a special injury, therefore they’re direct claims.
3. TX/IN: They’re derivative claims, but the court has discretion to treat
them as direct suits, depending on the circumstances (as long as other
shareholders, creditors, etc. wouldn’t be hurt by making it direct).
e. Delaware once approved the “special injury” test, but in Tooley v. Donaldson (pg. 733),
the SC rejected it and said the analysis must be based solely on two questions:
i. Who suffered the alleged harm—the corporation or the suing stockholder?
ii. Who would receive the benefit from the recovery or other remedy?

f. Demand on the Board of Directors

i. Johnson v. Steel, Inc.


1. To be a π in a derivative suit, the π must have either made a demand on
the Board or show that making such a demand would be futile.

74
2. In this case, π argued a demand would be futile because the perpetrator
and his wife are on the Board, along with another self-dealer.
3. Aronson Test: Does the complaint raise a reasonable doubt that the
business judgment rule would protect the board’s conduct?
a. In this case, there is obvious self-dealing, so there is reasonable
doubt that the business judgment rule would apply.
b. Aronson says there are three ways to show reasonable doubt that
BJR applies:
i. Directors are self-dealing (interested in/standing on both
sides of the transaction)
ii. Directors breached their duty of care.
iii. The substance of the transaction is so egregious it gives
rise to an inference that the directors breached their duty
of loyalty or care.
4. The shareholder π’s attorney is also a source of concern.
a. Lots of plaintiffs’ attorneys have a history of clients who own
one share of stock in every major company. Then, they keep an
eye out for corporate malfeasance and launch derivative suits.
5. The Aronson standard is generally the law everywhere.
a. It’s a tough standard because the π hasn’t had a chance to take
discovery yet.
6. The demand rule and futility rule aren’t an exhaustion requirement.
a. If a disinterested board bars you from bringing the derivative
suit, the lawsuit is over.
b. If you have to make a demand, and the Board complies with the
duty of care in turning you down, you’re barred. In effect, you’re
out of luck if a demand is required.
7. In Texas and other model act jurisdictions, there is never a futility
exemption – you have to make a demand in every case.
a. Then, a court will decide if the Board was right to bar the suit
(using the same Aronson factors).
b. In other words, TX has an exhaustion requirement – a π must
exhaust internal corporate remedies.
8. If you’re exempt from making a demand, you should never make one.
a. If you make a demand to an interested board, they’ll make an
independent special litigation committee and bar the suit, then
you’ll be out of luck.
b. The BJR protects the special litigation committee’s decision in
some states.
c. Other states never respect the SLC’s decision if the committee is
created by an interested board.
d. Another group of states, led by Delaware, has the option of
deferring to the SLC or not.
9. Disinterested boards or SLCs can consider litigation costs and business
costs when making a decision as to whether to do a derivative suit.

g. Demand on the Shareholders

75
i. A minority of states will require that a π make a demand on the shareholders if a
demand on the board will be futile. Exceptions:
1. If the majority of shareholders are interested.
2. If the decision couldn’t be ratified unless all the shareholders vote in
favor.
3. If the corporation has so many shareholders that making a demand would
be prohibitively expensive.

h. Shareholder Standing

i. Bangor Punta Operations, Inc. v. Bangor & Aroostook Railroad Co.


1. Amoskeag couldn’t have brought this suit as a derivative suit because of
the contemporaneous ownership rule and the tainted shares rule.
a. Amoskeag didn’t own the shares at the time of the wrongdoing,
and he bought the shares from the shareholder who committed
the mismanagement and self-dealing.
2. Why have the contemporaneous ownership rule?
a. Amoskeag bought the shares at the post-stealing price. It’d be a
boondoggle to then compensate Amoskeag for the effect of the
stealing.
b. Exception: If the shares were obtained by inheritance OR the
wrongdoing continued after the shareholder bought the shares.
i. What constitutes a “continuing wrong” is a murky area
of the law.
3. This is a direct action rather than a derivative suit. Why does the court
bar the suit?
a. For all the same reasons Amoskeag can’t bring a derivative suit,
it’d be inequitable to award damages to him on a direct basis.
4. Why not allow the company to bring the suit, then award damages only
to the innocent minority?
a. The court didn’t do that because Amoskeag would have dropped
the suit – he just wanted the boondoggle.
5. Another major standing requirement: The π must be an adequate
representative of the shareholders, just like a named plaintiff in a class
action.
a. Turns on whether the π has any conflicts with the Corp, but court
will entertain any other reason

i. Individual vs. Corporate Recovery


i. What is better about giving the recovery to the company rather than an
individual?
1. It protects creditors – they can’t collect unless the corporation has
money.
2. The court shouldn’t be involved in the decision to declare a dividend to
shareholders.
3. It’s also the formalistic logic of the derivative suit to have the
corporation, not shareholders, recover.

76
ii. The court always has the equitable power to order a pro rata recovery for the
injured shareholders.
1. Bangor Punta is the poster child for courts exercising equitable power.

iii. Glenn v. Hoteltron Systems, Inc.


1. The court pays the company because it’s fair to do so.
2. S supposedly usurped business opportunity. He got 100% of the gains,
but only 50% of the loss. Extra benefit of $180K.
3. 2 choices:
a. Put $360K back into the company. (Costs $180K; would take
100% loss and only 50% gain.)
b. Give $180K to the plaintiff. (Would cost $180K. Only 50%
loss.)
4. No economic differences as a matter of value.
5. Plaintiff argues that if you put the money back in the company, S gets
1/2 of the gain and that’s not fair.
6. Court rejected this proposition. As long as it’s proportional, it doesn’t
matter. Only needs to pay back what he stole. He can retain any fair
benefits
a. Also, should put back in the company for policy reasons:
i. Creditors
ii. Rules for paying dividends should be followed for
disbursement.
iii. Derivative suit = shareholder suing on behalf of the
company.
7. Rule: Where there is no economic difference, we usually pay back to the
company. But, if the creditors are not harmed and there is an equitable
reason why it makes more sense to pay to the minority shareholders, the
court can. E.g., Bangor Punta. (New majority shareholder paid post-
stealing price. Otherwise, would be substantially over-penalized.)

j. Paying Shareholders’ Costs and Attorneys’ Fees


i. If there’s a substantial benefit to the corporation, then π can get attorneys’ fees
and costs paid.
1. Even if the judgment isn’t in π’s favor, π can get attorney fees paid if the
corporation reforms itself to its own benefit (see Tandycrafts).
ii. Many states (not DE) require security for expenses, where π must put up money
for the corporation’s litigation expenses. If the Δ wins, Δ keeps the security.
iii. In Texas, there’s no requirement for security for expenses, but a victorious Δ can
petition for the π to pay Δ’s expenses if the court finds the case lacked merit.

k. Settlements
i. Settling is no longer solely the province of the shareholder-π. The court must
decide whether the settlement is fair, and the other shareholders get notice and an
opportunity to intervene and keep the suit alive.

77
ii. A 2nd Circuit decision says the corporation can settle the suit out from under the π
aby settling independently, even without the corporation’s permission (Wolf v.
Barkes).

78
IV. Limited Partnerships
A. Overview
1. A limited partnership must have at least one general partner and at least one limited partner
2. The general partner is generally liable for all of the partnership’s obligations and limited
partners generally aren’t.
3. Management responsibilities can be configured however you want through contract (Limited
Partnerships are creatures of contract).
4. Ditto for financial obligations.
5. The flexibility of contract is an advantage of the limited partnership.
6. You can’t create a limited partnership by accident. You must file a certificate with the
Secretary of State.

O. Defective Formation
a. Direct Mail Specialist, Inc. v. Brown
i. Π thought he could sue the limited partners because the partnership didn’t file a
certificate with the secretary of state, as required by statute.
1. The partnership also didn’t list who was a general partner and who the
limited partners were.
ii. The standard for whether a limited partnership certification is sufficient is
whether it substantially complies with the statute.
1. Filing with the wrong office is a big problem because a third party would
expect the certificate to be on file with the secretary of state’s office.
2. If nothing’s on file with the Secretary, then a third party can assume it’s
a general partnership.
iii. What if they’d filed in the right place, but still hadn’t explained who the general
and limited partners are?
1. This is also likely a problem, although you could argue that it gave them
sufficient notice that not all of the partners are liable.
iv. Under RULPA § 201, you only have to list the name of the limited partnership,
the address, the names and addresses of general partners, and the latest date the
partnership is to dissolve.

P. Entity Status
a. Currier v. Amerigas Propane, L.P.
i. Π’s employer is Amerigas, Inc., the general partner. He sued the general partner
in a workman’s comp proceeding.
1. Πs like workman’s comp because the payments are certain and standards
of proof are much lower.
2. Δs like it because the payments are much lower than in a normal tort
proceeding.
ii. The π in this case sues the limited partnership for the rest of his damages that
weren’t covered by workman’s comp.
iii. The court says that the limited partner is not a separate entity for the purposes of
substantial obligations.

79
1. This has to do, in this case, with the point of workman’s comp. The π
gets paid in exchange for giving up the right to sue the employer
anywhere else. Allowing the limited partner to be sued – and have the
general partner be on the hook for the damages – would defeat the
purpose of workman’s comp.
2. The real question of this case was whether the partners were separate
entities for the purposes of workman’s comp. The Limited Partnership
Act doesn’t really have anything to do with it.

Q. When is Limited Liability Lost?

a. The Control Rule


i. Absent some piercing of the veil, shareholders are never liable for the obligations
of the corporation beyond their investment – no matter the level of control
exercised.
ii. Not so for limited partners under the old ULPA.
iii. Holzman v. De Escamilla
1. Π is a trustee in bankruptcy.
2. The case is decided under the 1916 ULPA.
a. If you exercise control over the partnership, you lose your
limited liability.
3. What was the point of the old control rule?
a. Limited partnerships have to have at least one general partner:
today the general partner can be a limited liability entity such as
a corporation.
b. The point of the old rule was that third parties would assume that
any partner engaging in control is a general partner.
4. In this case, the limited partners decided what crops to plant, etc., so they
engaged in control and lost their liability. They also could withdraw
funds.
5. The Δs’ lawyers should have advised them about what level of control
would result in a loss of limited liability. Veto power is OK, but
affirmative day-to-day control is too much.

b. The Control Rule was narrowed over time.


i. Gateway Potato Sales v. G.B. Investment Co.
1. The Δs, limited partners, exercised control in many ways.
a. They chose the accounting method, chose which accountants to
use, decided which debts to pay, etc.
b. Under the 1916 rule, there’s no question this is control.
2. Mr. Ellsworth hated G.B. Investments, so he was perfectly willing to
give the whole list of things Δs did to exercise control.
3. The 1976 RULPA holds that control on its own isn’t enough to lose
limited liability.
a. The control must also be substantially the same as the control
exercised by general partners OR a third party has actual
knowledge of the limited partners’ controlling activities.

80
b. The test was narrowed because control is only a problem if it’s
misleading.
ii. The 1985 RULPA narrowed the rule still further: A limited partner must exercise
control and the third party must have a reasonable belief that the limited partner
was a general partner based on the limited partner’s conduct.
1. In Gateway Potato, there is definitely reasonable belief that the limited
partner is a general partner because Δs were held out as Potato’s backers.
a. However, there was no third party contact with the limited
partner to show that the belief stemmed from the limited
partners’ conduct.
b. The court says such contact is required, but the statute doesn’t
require this.
i. The statute, by its text, requires only that the limited
partners’ conduct is the source of the belief.
iii. The 2001 ULPA eliminates the control rule altogether.
1. This makes things more efficient. The people listed on the certificate
with the secretary of state are the only ones who are liable.
iv. Almost no jurisdictions have adopted the 2001 ULPA, though. The 1985 RULPA
is the law for this course.

R. Fiduciary Duties in Limited Partnerships

a. Gotham Partners, L.P. v. Hallwood Realty


i. The directors engaged in an odd lot tender offer, a reverse split, and a unit option
plan.
1. A reverse split reduces unitholders’ number of units, which results in an
increase in the unit price. (e.g. – someone with 100 units would have
66.6666 after the reverse split).
a. This stems from an arbitrary belief that $50 is the ideal unit price
– even though the underlying value of the partnership hasn’t
changed.
2. This results in odd lots (a lot of fewer than 100 units).
3. Odd lots are not as desirable as even lots because the brokerage fees are
higher.
4. The unit option plan allows the managing partners to buy up some of the
lots – the idea is to incentivize the management to do well.
ii. HGI, the general partner of the partnership, has made a tender offer to buy up the
odd lots.
1. Unless there are contractual modifications, the entire fairness test applies
to this transaction.
iii. The partnership agreement requires a process that’s similar to the entire fairness
test:
1. An audit committee must review the deal for fairness (fair dealing), and
the price must be at market value (fair price).
2. Not quite the same thing as entire fairness, but it’s pretty close.
iv. DRULPA allows modification, but elimination of fiduciary duties.

81
1. RULPA says that a general partner must abide by general partnership
rules, including what you can’t do to modify fiduciary duties (RULPA
403):
a. RUPA 404(b) says what the duties of a general partner are.
b. RUPA 103 says what you can’t change about the fiduciary
duties.
i. RUPA 103(b) says you can’t eliminate the duties of
loyalty and care.
2. ULPA (2001) jettisons the concept of linkage, so you don’t have to make
the above journey back into RUPA.
b. Limits of the Partnership Agreement: Labovitz v. Dolan
i. Dolan withheld tax indemnification from the limited partners, forcing them to
pay huge tax bills without the liquidity from the partnership’s income.
ii. Most non-corporate entities prefer pass-through taxation. They money is taxed
only once.
iii. Withholding tax indemnification is an effective freezeout technique.
1. The LPs take the freezeout deal (2/3 of book value), then sue.
iv. Wilkes says that partners have fiduciary duties to each other.
v. There’s no implied elimination of fiduciary duties – they must be eliminated
explicitly.
vi. Dolan’s partnership agreement gives absolute authority to the general partner to
distribute or not distribute funds.
1. However, the agreement doesn’t disclaim fiduciary duties of good faith
and fair dealing.
vii. The court finds the agreement and Dolan’s interpretation of it to be manifestly
unreasonable and, therefore, an illegal modification of fiduciary duties.

S. Fiduciary duties when the general partner is a corporation: In Re USACAFES, L.P.


a. The πs complain that the general partner sold assets to another entity at too low a price.
b. The Wyly brothers were directors of the corporate general partner – they got a kickback
of $15-$17 million (which was a breach of loyalty).
c. The πs also sue the board members who didn’t benefit from the transaction for breach of
their duty of care.
i. No market check, no auction, and no financial analysis from an investment
banker.
d. The Wylys say they can’t be sued for breach of fiduciary duty because they are not the
general partner, just the directors of the corporation.
e. Holding: The directors of the corporate general partner owe a duty to the limited
partners, under the same theory as for when there is a corporate trustee of an estate and
the directors of the corporate trustee have a duty to the estate.
i. This is a way of ignoring the separate entity status of the corporation.
f. Could the Wylys be sued for something other than breach of duty?
i. Perhaps on an aiding and abetting claim. The corporation was stealing and the
directors were helping.
1. However, the mens rea requirement may be tricky because it requires at
least recklessness.

82
2. The mens rea on a breach of duty claim, however, is lower: gross
negligence.
ii. Also, for self-dealing, the entire fairness test applies and there’s no mens rea
requirement.
g. This is a close call, because aiding and abetting is not a substitute for a direct action
against the directors. Also, the corporation is entitled to be underfunded, and the outside
world would have to deal with it. So, theoretically, the limited partners should also have
to take that risk.

T. Duties of Limited Partners


a. RULPA (1985) does not address the fiduciary duties of limited partners, so according to
§1105, General Partnership law applies – where all parties owe duties to each other.
However, this is an awkward fit for the limited partnership context, where limited
partners don’t exercise the kind of control that necessitates fiduciary duties.
b. KE Property Management Inc. v. 275 Madison Management Corp.
i. KE wanted to kick out 275 MC as managing general partners pursuant to the
partnership agreement.
1. The agreement requires a 25% vote of limited partners and that the
managing partner have committed fraud or willful misconduct.
ii. However, there was an issue as to whether removing the managing partner for the
purpose of preventing bankruptcy protection was a breach of KE’s fiduciary
duty.
iii. Holding: The Court reasons that when limited partners exercise control of a
partnership’s governance, they owe fiduciary duties.
1. This principle is imported from corporate law (see Atlantic v. Smith,
where a minority shareholder with veto power owes a fiduciary duty in
exercising that veto).
iv. MC argues that KE was exercising its control in bad faith – KE wanted to ensure
there are no bankruptcy protections so that Kawasaki lenders can collect the
money it was owed.
1. The K Lenders can’t exercise their security interest automatically if the
partnership is in bankruptcy.
v. If KE, as managing general partner, keeps the partnership out of bankruptcy for
the purpose of enriching the K Lenders, then the partners can sue KE for breach
of their fiduciary duty as a general partner.
vi. Fiduciary duties of Limited Partners are the same as fiduciary duties of minority
shareholders.

U. Ownership Interests and Transferability


a. The default rule is that financial rights in a limited partnership are transferable while
management rights are not (RULPA 101(10), 702). Under RULPA 704, an assignee of a
partnership interest has the right to become a limited partner “if and to the extent that (i)
the assignor gives the assignee that right in accordance with authority described in the
partnership agreement, or (ii) all other partners consent.”

83
b. Entity General Partners: This raises an interesting issue, because corporate shares are
freely transferable property, yet the corporation retains its identity even if all of its shares
are sold to someone else and the control changes hands. Such a result could adversely
affect the interests of limited partners, who had no say in the transfer of control – in
contravention of the typical rules of partnership.
c. In Re Asian Yard Partners
i. Default rule is that there are no new partners in a limited partnership without
unanimous consent. This rule can be changed in the partnership agreement.
ii. In this case, the general partner is a corporation and they’re unilaterally selling all
of the corporation’s stock.
iii. The corporation will remain the same, but someone else will own and control it.
iv. The partnership agreement says you can’t transfer your interest in the partnership
“directly or indirectly.”
1. The point of this kind of restriction is so that the limited partners can
know who’s in charge of the partnership. They don’t want strangers to
take over without the limited partners’ knowledge and consent.
2. This kind of clause doesn’t prohibit transfer by operation of law unless it
says so explicitly.
3. Transfer restrictions are valid as long as they’re reasonable.
v. Some courts will invalidate a transferability restriction that is too universal. You
can’t veto a transfer without cause.
vi. The statute says that it requires unanimous consent to make a new general
partner.
1. This says the legislature thinks it’s OK for there to be an unlimited
consent requirement to change a partnership’s management.
2. If this is so, then it makes sense there’d be no reasonableness
requirement for a transferability restriction in the partnership context.
d. Charging Orders
i. RULPA 703 provides for a charging order procedure where the court may charge
the partnership interest of the partner in order to pay the unsatisfied amount of
the judgment a creditor seeks. In other words, creditor collects indebted partner’s
checks.

ii. Baybank v. Catamount Construction, Inc.


1. What do you do when you want to collect against someone whose assets
are all tied up in a partnership?
a. You first get a charging order (RUPA 504).
b. Then, you foreclose on the partners’ income stream from the
partnership.
2. If this doesn’t work, you dissolve the partnership. RUPA 504 says a
creditor is an assignee. RUPA 801(6) says an assignee can dissolve the
partnership.
3. Linkage applies in this case, so the RUPA comes in to allow creditors to
foreclosure on partnership interests.
4. The ULPA (2001) allows for charging orders, but not dissolution, so it’s
possible that this is a clarification of RUPA and that the RUPA was
intended to disallow dissolution by a creditor.

84
5. The court finds that dissolution by a creditor was explicitly not permitted
in the RULPA, but foreclosure wasn’t mentioned, so foreclosure is
permitted.
a. Policy: Foreclosure is much less intrusive on the other partners
than dissolution.
6. The comments to RULPA seem to confirm the court’s holding that
dissolution isn’t linked to RUPA. § 801 cmt says 801 collects every
instance when dissolution is OK, and creditors aren’t included.
7. The language of RULPA wasn’t clear, but policy, comments, and
subsequent legislative action point in the court’s favor.

V. Exit Rights: Dissociation and Dissolution


a. Dissociation:
i. RULPA § 402 specifies the events of withdrawal for a general partner,
including: voluntary withdrawal, removal, and bankruptcy. RUPA § 602 allows a
general partner to withdraw at any time by giving written notice to the other
partners, but if withdrawal violates the partnership agreement, the limited
partnership may recover damages. Pursuant to § 604, a withdrawing partner
(general or limited) is entitled to receive any distribution provided for in the
partnership agreement.
ii. RULPA § 603 allows a limited partner to withdraw under circumstances
specified in a written partnership agreement. If the agreement is silent, “a limited
partner may withdraw upon not less than six months’ prior written notice to each
general partner.” § 604 provides for a “fair value” buyout.
b. Dissolution:
i. Under RULPA § 801, a limited partnership is dissolved (1) at the time specified
in the certificate of limited partnership; (2) upon the occurrence of events
specified in a written partnership agreement; (3) upon the written consent of all
partners; (4) upon an event of withdrawal of a general partner under § 402; and
(5) by the entry of a decree of judicial dissolution under § 802.

c. Obert v. Environmental Research & Development Corp.


i. Facts:
1. The partnership agreement allowed removal of a general partner with a
66% vote.
2. 74% of the partners voted for removal by proxy.
3. The partnership agreement allowed for proxy votes under corporate law,
but was silent as to whether a meeting was required.
ii. The court finds that the reference to corporate law was only intended to
determine the requirements for being a proxy:
1. Must be in writing;
2. Must be signed;
3. To be irrevocable, it must expressly state that it’s irrevocable;
4. Must be coupled with an interest.
5. (Ringling Bros, DGCL § 212)
iii. The meeting would be pointless because 74% were in favor of removal.

85
iv. § 14 of the partnership agreement has a laundry list of things that require a 66%
vote.
1. When they drafted § 18 requiring a unanimous vote to appoint a
successor general partner, the drafters must have forgotten that § 14
already required only a 2/3 vote.
2. *Always be on the lookout for specific provisions that contradict general
provisions.
v. The court doesn’t apply the principle that specific provisions trump the more
general provision.
vi. If a limited partnership doesn’t have a general partner, then there is no limited
partnership – it’s a statutory requirement.
vii. Is there a difference between admitting a general partner and appointing a
successor general partner? No.
viii. For every principle of statutory construction, there’s an equal and opposite
principle.
ix. In this case, the court applies the principle that when two sections of statutory
language deal with similar things, they should be read similarly (in pari materia).
x. RUPA doesn’t forbid contracting around the rules regarding appointment of new
partners.
1. Therefore, through linkage, one could argue that a limited partnership
can alter the rule regarding unanimous consent for general partnerships.

86
87
V. Limited Liability Companies
A. Overview
(a) The chief advantage of an LLC over a corporation is you can contract any rule you want
– there are no regulatory rules.
(b) The advantage over a general partnership is limited liability.
(c) The advantage over a limited partnership is there is no risk of being held liable if you
exercise too much control.
(d) An advantage over corporations is Pass-Through Taxation:
i. The IRS used to take the view that to get pass-through taxation, you had to
disclaim at least two elements of corporations.
ii. Now it’s a “check the box” regulation. As long as the LLC isn’t publicly traded,
you can choose what kind of tax treatment you’ll get.
(e) 100 years from now, all but the largest publicly traded companies will be LLCs.
(f) The biggest reason all closely held companies aren’t LLCs now is because of the lack of
certainty in the law.
i. You have to have a very comprehensive contract because the statutes don’t cover
everything.
ii. Case law is also not well developed because LLCs are relatively new.
(g) Almost all states have their own LLC statutes and haven’t adopted the ULLCA.
i. Reverse of the usual situation for previous statutory business organizations
(h) As of now, there are gaps in LLC coverage in state regulations. So, for each case, courts
must decide whether an LLC is more like a corporation or a partnership for any given
purpose.
i. E.g. – In Oklahoma, a constitutional provision prohibits a corporation from
owning a liquor license – it must be owned by a full-liability entity. So, an LLC,
as a limited-liability arrangement, can’t own a liquor license.

B. Formation
(a) LLCs are formed by filing a document called the “articles of organization.” It typically
has only basic information about the LLC. In Delaware, it’s only required to have the
name and address of the LLC as well as the name of a registered agent for service of
process.
(b) The LLC agreement has the real details on how the LLC will be run. It’s not a public
document. It contains specifics on rights, duties, and obligations of the LLC’s members
and managers and on the operation of the LLC as a whole.

C. De Facto LLCs
(a) Well settled in the corporate context, but a new question for LLCs.
(b) Duray Development, LLC v. Perrin
i. The contract was signed by the LLC a month before the LLC was officially
formed.
ii. Three factors of de facto corporation:
1. Acting under a valid statute
2. Bona fide effort to incorporate

88
3. Presented self as a corporation
iii. Corporation by estoppel allows limited liability status when a third party
understands his contract to be with a corporation.
iv. What’s the point of de facto corporate status? They don’t want to punish people
who came pretty close to establishing a corporation.
v. If you’re a DE LLC, you’re protected both by de facto corporation doctrine and
corporation by estoppel doctrine.
vi. The problem with LLCs is that you’ll have to litigate this issue, even though
you’ll probably win if you want de facto/corporation by estoppel status.

D. The Role of Contract


(a) LLCs, to a greater extent than limited partnerships, are creatures of contract. LLC statutes
typically contain few or no default rules on meetings, quorums, and notice – it’s all up to
the contracts.
(b) Elf Atochem North America, Inc. v. Jaffari
i. Jaffari provides the chemicals and Elf provides the access to international
markets.
ii. Jaffari started finding his own clients, so he started running the LLC into the
ground and providing sub-standard chemicals.
1. Jaffari was cutting Elf out of the business, which is illegal for partners
per Meinhardt v. Salmon.
iii. Is Elf’s suit a direct or derivative suit?
1. It could be derivative in every state in the country because the harm was
done to the LLC and a recovery would benefit the LLC.
iv. Jaffari argues that the LLC didn’t sign the contract, so he’s not bound by
anything.
v. The court finds that the LLC is bound under the LLC agreement because all the
members have signed it.
1. This is an aggregate theory. The entity status of the LLC is context-
specific.
vi. Even if the LLC weren’t bound, the arbitration clause binds members, and a
member is the one who brings a derivative suit.

E. Member-Managed versus Manager-managed LLCs


(a) Member-managed LLCs are like general partnerships in terms of control.
(b) Manager-managed LLCs can be like corporations or like limited partnerships.
(c) The default rule in Delaware is to have a member-managed scheme (DLLCA 18-402).
i. Another default rule is that members vote in proportion to their share of
contributed capital (DLLCA 18-503).
(d) About half of states say the default is member-management while the other approximate
half says the default is manager-management.
(e) In Texas, the certificate is supposed to say which kind of management there’ll be or else
the LLC is improperly formed
i. Rags doesn’t know what would happen if SecState of TX accidentally filed an
LLC cert. lacking this designation (would presumably need a default rule)
(f) Kahane v. Jansen

89
i. The most important element of the malicious prosecution case here is that the suit
was brought without probable cause.
ii. Kahane sues Jansen for malicious prosecution because she sued him for breach of
fiduciary duty and malpractice, which ruined his law career.
iii. Kahane was appointed as manager of Jansen’s LLC, probably just to manage
loan documents.
iv. Lawyers’ relationship to the organization that hires them is an important thing to
keep in mind:
1. In a corporation, SHs can’t sue the corporation’s lawyer for malpractice.
The lawyer’s client is the corporation – only the corporation can sue the
lawyer. The lawyer doesn’t represent the shareholders.
2. For a partnership, the partnership’s lawyer also represents the partners.
This is despite the partnership’s entity status because the partners are
fully liable for the partnership’s liabilities.
v. According to Jansen, at meetings with TATA/Tucker, Kahane was introduced as
and held himself out as the attorney for the LLC and did not disclose his
representation of TATA and Tucker
1. Did Kahane have duty to represent the interests of the LLC and its
members equally, and to disclose his potential conflict of interest?
2. In the first suit, the court held that an LLC is more like a corporation than
a partnership, so the court held that Kahane didn’t owe a duty to the
LLC’s members.
vi. However, this wasn’t a clear question – the law was unsettled, so the court had to
look to the law of corporations and partnerships to find the answer.
1. Therefore, there was sufficient “probable cause” such that Kahane can’t
win his suit for malicious prosecution.
iii. NOTE: **From Shymanski v. Coventz: Default rule: Services aren’t compensated
or counted toward the capital account unless there’s an express or implied
agreement otherwise.
vii. Lesson: When you represent an entity, always make it clear to the
members/shareholders that you represent the entity, not them.’

F. Authority in the LLC Context


(a) Taghipur v. Jerez (Utah)
i. The members are mad at Jerez because he took out a loan in the LLC’s name,
then absconded with the money. He used the LLC’s real property as collateral.
ii. Jerez didn’t have express or implied actual authority to borrow money because
the operating agreement didn’t allow it.
iii. In the partnership context, apparent authority to borrow depends on whether:
1. The borrowing is in the regular course of business; and
2. It’s typical in that type of business in that geographic location.
3. Cite to Harvard(?)
iv. Just because the operating agreement doesn’t bestow actual authority doesn’t
mean an LLC manager lacks apparent authority.
v. The Utah Real Estate statute says that if it’s signed by a manager, a document
conveying real estate is valid no matter the authority of the signer.

90
1. This is to provide certainty as to who owns what.
2. In this case, the question is whether the real estate statute trumps the
operating agreement.
vi. A Utah legislator would have wanted the real estate statute to take precedence
because of the importance of real estate and its perceived greater value than LLC
flexibility.
1. Therefore, the court finds Jerez’s authority is irrelevant – the Real Estate
statute controls.

W. Limited Liability: The Scope of Limited Liability


a. Water, Waste & Land, Inc. v. Lanham
i. Mr. Lanham made an oral agreement with Mr. Clark. Lanham gave Clark his
business card, which said “PII” on it.
ii. Authority Review: according to 3RA § 2.06, an undisclosed principal may still be
held liable to a third party who justifiably is induced to make a detrimental
change in position, even if the agent lacked actual authority to act on behalf of
the principal, so long as the undisclosed principal had notice of agent's conduct
and that it might induce the third party to change its position, and the principal
did not take reasonable steps to notify the third party of the facts.

1. A principal is liable on a contract if the agent had any kind of authority


to make it.
2. The third party is liable unless the principal or agent knew the third party
wouldn’t have agreed if he’d known the identity of the principal.
3. The agent is always liable unless the principal is fully disclosed.
iii. So, in this case, the question is whether the business card disclosed the principal.
iv. The card only said “PII,” which is not close to fully disclosing the principal.
v. The LLC statute says that a member isn’t liable merely because he’s a member.
He can be liable for any other reason.
vi. In this case, Mr. Lanham isn’t liable because he’s a member – he’s liable because
he’s an agent whose principal isn’t fully disclosed.
vii. The court says the LLC statute would protect the members from a veil piercing
action.
1. This isn’t obviously right. The whole point of veil piercing is to hold
members of a limited liability entity responsible for the entity’s liabilities
when the individuals are using the entity structure to cheat creditors.

b. Kaycee Land & Livestock v. Flahive


i. The controversy turns on whether or not the Wyoming LLC statute permits veil
piercing.
1. There’s a perfectly good narrow reading of the statute that allows veil
piercing.
ii. It’s harder to pierce the veil of an LLC – formalities mean very little (in most
states). Some states, like Texas, place no stock in formalities at all in the LLC
context.

91
c. VGS v. Castiel (Delaware)
i. It’s a manager-managed LLC.
1. Mr. Quinn is not a member (i.e. – a unitholder), but he is on the board of
managers.
ii. Sahagen probably has good motives in freezing out Castiel.
iii. Quinn, who was Castiel’s appointee, and the company’s employees and directors
testify that Castiel was doing a terrible job.
iv. The LLC agreement doesn’t say what vote is required to approve a merger.
v. The DLLCA doesn’t have a default rule regarding votes required to approve a
merger.
1. This is the disadvantage of the LLC form.
vi. Sahagen owns a 25% unit stake. The LLC agreement gives him a percentage of
board seats that is less than his ownership stake.
1. If there were 8 board members, he would just have one member because
2 would be 25% of the board.
vii. 8.01(c) of the LLC agreement gives Mr. Sahagen a veto on mergers.
1. If it took a unanimous vote to do anything, Sahagen would already have
veto power over everything.
2. This implies a majority vote is sufficient for mergers, provided Sahagen
consents.
viii. The LLC agreement also allows a 2/3 membership vote to approve a dissolution.
This implies that it requires a unanimous vote for a merger, through expressio
unius.
ix. When drafting an LLC agreement, remember to include all of the operational
rules because there are no defaults.
x. The strongest argument is that the court should look to the corporate context. The
LLC agreement and statute are unclear, so the court should have reasoned that
the drafters meant to default to corporate law, which is the closest analogue to a
manager-managed LLC.
xi. Quinn and Sahagen got written consent from the members to push the merger
through.
xii. DLLCA § 18-404(d) says a written consent can be signed by managers who
have enough votes to win. No meeting necessary.
1. If they had had a meeting, they’d have to give notice to Castiel and he
would have used his 75% stake to kick out Quinn and get a director he
can trust to nix the plan.
xiii. The entire logic of the LLC agreement is that Castiel has total control except as
to matters over which Sahagen has a veto.
xiv. Holding: The court says that Sahagen has breached his duty of loyalty by
making an end run around the whole purpose of the LLC agreement.
1. This is inconsistent with Nixon v. Blackwell, where the Delaware
supreme court said that, where a shareholder fails to properly contract in
such a way as to protect himself, then the courts won’t create fiduciary
duties to protect the unwary.
2. This case is also inconsistent with Farnsworth.
xv. This is the leading case for the proposition that members of an LLC owe special
duties to one another.

92
d. McConnell v. Hunt Sports Enterprises (Ohio)
i. Facts:
1. Hunt was mad at McConnell for starting up an NHL franchise out from
under the LLC he was a part of.
2. McConnell was probably saving the NHL franchise for Columbus
because the LLC was going to let the opportunity slip away because they
couldn’t reach an agreement with the city.
3. Hunt is mad because McConnell undercut Hunt’s bargaining power
because he would have come up with a deal at the last minute on better
terms had McConnell not accepted the offer first.
a. Therefore, although McConnell’s motives were good, Hunt
believes McC was misguided and also illegally competed with
the LLC.
4. However, § 3.3 permits LLC members to engage in “any other business
venture” whether or not it competes with the LLC – plain text allows
McC’s gambit.
5. Courts do not favor waivers of fiduciary duties by implication.
a. § 3.3, if read to allow competing for hockey, is a waiver of the
duty of loyalty.
6. In both the corporate and partnership context, you can’t eliminate the
duty of loyalty.
7. Despite this, the court found the duty of loyalty had been waived (don’t
forget this is an elected state court judge
e. Anderson v. Wilder (re: LLC freezeouts)
i. The minority unitholders are upset because they were expelled from the LLC
without cause and for a $150/unit buyout price. Their contract allowed for both
things.
ii. VGS v. Castiel makes it clear that members of an LLC have a fiduciary duty to
other members (in Delaware).
iii. Wilkes says that closely held corporate shareholders owe each other a fiduciary
duty (in Massachusetts).
iv. In Delaware, there’s no special duty shareholders owe each other in a closely
held corporation (Nixon v. Blackwell).
1. However, Delaware recognizes a fiduciary duty owed by majority
shareholders to minority shareholders to abstain from self-dealing
(Sinclair Oil).
v. In this case, the $150 buyout price isn’t fair because the units sold for $200 on
the open market.
vi. Are members of an LLC fiduciaries for one another in the Tennessee context?
1. LLCs are creatures of contract – maybe that means there are no fiduciary
duties.
vii. Δ argues that they expelled the πs in good faith because the LLC needed money,
and the πs wanted to engage in bad business practices.
viii. Π said they were expelled so the LLC could buy the πs’ units back cheap, then
sell them at a higher rate.
ix. This could be illegal because the Δs are enriching themselves at the expense of
the πs, which is self-dealing.

93
x. May be legal because the πs contracted for the expulsion-at-will provision. See
also Gallagher, where the court said “tough” to a π who was fired so the
corporation could buy back his shares cheaply – his employment contract said he
could be fired for any reason or no reason.
xi. Holding: If the Δs expelled the πs to steal the value of their units, it’s illegal
despite the contract saying they could be expelled for any reason or no reason.
1. You can tell they weren’t really expelling the πs for a business reason
because they made cash disbursements as soon as they were gone.
f. K.C. Properties, Inc. v. Lowell Investment Partners, LLC
i. The πs complained because LIP sold a property without cutting the πs (KC
Prop.)in on the profits.
ii. LIP was a 51% member of Ozark LLC and KC was a 49% member. PMS was
the manager and PHR was the owner of the property.
iii. Why can the πs sue PHR? If they’re not fiduciaries of each other, it will have to
be a suit for breach of contract.
1. The members of PHR are also members of PMS. KC is saying, “You’re
double-dealing on us” and ignoring the separate entity status.
iv. How does PHR get sued for breach of fiduciary duty?
1. USACafes says that the corporation directors who ran the corporation
that was the general partner of a limited partnership owe a fiduciary duty
to the limited partners.
2. Entities that owe a fiduciary duty also have fiduciary duties extended to
the people in control of the entity.
3. If USACafes is correctly decided, then there’s no question that PHR
owes a duty to KC.
v. The court posits an agency idea that’s ridiculous. There’s a fully disclosed
principal, therefore the agent isn’t going to be bound by the contract.

X. Ownership Interests and Transferability


a. Ownership interest in an LLC entitles a member to:
i. (1) Financial rights: receive distribution and share in the profits/losses
ii. (2) Management rights: participate in the management and control
b. Like partnerships, all LLC statutes provide that an assignment of an LLC ownership
interest transfers the member’s financial rights but NOT his management rights
c. The statutes usually specify that an assignee can acquire a member’s management rights
only with the consent of all of the non-transferring members
d. Rule: Unlike a shareholder but like a PARTNER, an LLC member cannot freely transfer
his full ownership interest
i. This limitation on transferability is simply a default rule that can be modified.

e. Spurlock v. Begley
i. There must be unanimous written consent for a full ownership stake to be
transferred. Therefore, the transferee did not get management rights when the
transferor gave him the stake without unanimous written consent.
ii. There is a constructive requirement for material performance before the other
party has to perform.

94
iii. Spurlock says he doesn’t have to pay for Begley’s 25% interest because Begley
can’t deliver on it.
iv. DLLCA § 18-703 allows for charging orders. 18-703(d) says that the charging
order is the exclusive remedy. However, it also calls it a lien, so it’s possible that
foreclosure is an option as well.
v. Compare: A creditor of a corporate shareholder can just seize the shares – they’re
like any other property. The buyer at the foreclosure sale gets the full rights of a
shareholder.
f. DLLCA § 18-703: Why does (b) call the charging order a lien?
i. It probably means that being a creditor of an LLC member is the same as being a
creditor of a limited partner.
ii. You can probably get foreclosure on the interest of the debtor member, but you
definitely can’t get dissolution.

Y. Dissociation and Dissolution


a. General
i. LLC statutes typically state that a member dissociates from a venture upon the
occurrence of certain specified acts (e.g. withdrawal, resignation, death, or
bankruptcy)
ii. With check-the-box regulations, there was no longer a tax-driven need for
member dissociation to trigger a dissolution of the LLC; states curtailed the
buyout and dissolution rights
iii. The easy liquidity provided by dissociation-triggered buyouts and dissolution has
been eliminated in many jurisdictions.
b. Lieberman v. Wyoming.com LLC
i. Lieberman is upset because the LLC fired him, so he wants to leave and
withdraw his share of the LLC.
ii. What does he get?
1. He just keeps his equity interest – he’s no longer a member of the LLC,
though.
2. He’s not required to sell and the LLC isn’t required to buy.
3. He’s exactly like a person with a charging order – a transferee.
4. He can’t sue for breach of fiduciary duty because he’s not a member
anymore.
iii. He’s in LLC Limbo, a terrible place to be. He’s in a worse position than a
minority shareholder in a closely held corporation because he can’t sue if the
LLC treats him badly.
iv. How could a required buyout be implied?
1. By looking to the law of partnership.
a. Dissociation is a partnership concept, and partnership law
requires dissociated members to be bought out.
b. Lieberman should have just sued for breach of fiduciary duty.
c. He should never have withdrawn as a member – he forfeit his
ability to sue for breach of fiduciary duty.
v. In Delaware, you can only leave an LLC if the operating agreement says you’re
allowed to leave.

95
1. If the agreement is silent, then you can’t withdraw.
vi. DLLCA § 604 provides a default buyout of dissociated members – but only if
the agreement allows for dissociation.
c. Dunbar Group, LLC v. Tignor
i. Tignor was commingling LLC funds with his own and other bad behavior.
ii. Dunbar wants dissolution – he wanted to sever ties with Tignor as quickly as
possible. He says it’s impracticable to carry on the business.
iii. The court says there’s no reason to dissolve because Tignor can’t interfere once
he’s expelled.
iv. The court put Tignor in LLC limbo, just like Lieberman.
1. He only has a right to receive checks. He has no management rights or
fiduciary duties owed to him. Dunbar has no duty to buy him out.
v. Arguably dissolution was the right result, but the court chooses a harsher
outcome.
d. LLC Limbo is a phenomenon unique to LLCs. Partnerships and corporations don’t have
such a thing where you retain equity interest, but no other rights and no market in which
to sell the interest.
e. Expulsion of an Owner
i. Walker v. Resource Development Co. LLC
1. Walker, a cousin of George HW Bush, meets the Three Bills and says
he’ll join the LLC and arrange for the financing of his rich contacts.
2. He’s also a drunk and a generally unpleasant person, so the Bills expel
Walker.
3. To ensure no one thought Walker had a relationship with the Bills, they
sold their interest to a different company.
4. There’s nothing in the LLC agreement allowing for expulsion of a
member. The DLLCA also doesn’t have a provision for expulsion, and
the court can’t order expulsion.
5. Delaware rule: Your membership stake is permanent unless the LLC
agreement says otherwise.
6. The court requires Walker to pay off his debt to the LLC before he’ll get
his 18% of the company – otherwise his interest will be reduced
proportionally.
7. The Bills should have sued Walker for breach of fiduciary duty.
a. To the extent they could prove damages, they could collect, but
it would at least put pressure on him to sell out.

96
VI. Insider Trading
A. Rule 10b-5
Rule 10b-5 Employment of manipulative and deceptive devices.
It shall be unlawful for any person, directly or indirectly, by the use of any means or
instrumentality of interstate commerce, or of the mails or of any facility of any national
securities exchange,
(a) To employ any device, scheme, or artifice to defraud,
(b) To make any untrue statement of a material fact or to omit to state a material fact
necessary in order to make the statements made, in the light of the circumstances under
which they were made, not misleading, or
(c) To engage in any act, practice, or course of business which operates or would operate as a
fraud or deceit upon any person,
in connection with the purchase or sale of any security.

a. Securities fraud elements:


i. Lie
ii. Materiality
iii. Scienter
iv. Reliance
v. Loss causation
vi. Damages
b. Insider trading happens when an insider has valuable information that no one else has. Is
it a bad thing when they trade on that information?
i. Yes. It increases the risk taken by every person buying and selling stock, so
they’ll demand higher returns to compensate. It would increase the cost of
acquiring capital.
ii. No. Others can watch what insiders do – the market will be derivatively
informed.
c. Chiarella v. United States
i. The first person convicted of insider trading: an employee of a printer who saw
paperwork revealing targets of companies’ tender offers
ii. The printer is printing forms required by the SEC for the bidder.
1. The target companies are left blank, but it’s not hard to figure out what
they are from the context.
2. This is valuable information because the targets are about to get a huge
offer (50% premium is the standard). It’s not just valuable – it’s
extremely valuable.
iii. After these offers are announced, the stock price will rise precipitously.
iv. The price doesn’t go all the way up to match the value of the tender offer because
of the risk it will fall through.
v. Chiarella has made $30,000 over 14 months by buying and selling stock of the
companies targeted for takeovers.
vi. Δ says he didn’t lie, so that element of 10b-5 and common law fraud isn’t met. A
lie is a misrepresentation, half-truth, or silence when there’s a duty to disclose.
1. Δ doesn’t have a duty to disclose because he doesn’t have a fiduciary
duty to either the bidder or the target.

97
vii. Why not create a statutory duty for everyone with insider information to disclose
it?
1. This would hurt the economy because there’d be weaker incentive to
identify and acquire information for the purpose of making a profit.
viii. The court says that mere possession of material nonpublic information does not
give rise to a duty to disclose it.
1. The SCOTUS rejects the idea that the line should be drawn when the
possessor of the information didn’t spend any time, money, or effort to
acquire the information.
2. They think state law should give rise to that duty – they won’t create the
duty at common law.
ix. The bidder doesn’t owe a duty to the target – it’s an arms-length transaction, so
the bidder and target have adverse interests.
x. The printer owes a duty to the bidder. Δ owes a duty to the printer. However, the
bidder, printer, and Δ don’t owe a duty to the target.
xi. The court believes that state law must create the duty, not federal securities law.
xii. The bidder wanted their offer to be kept secret until it’s announced – they don’t
want the target’s stock price to be driven up prematurely.
xiii. Misappropriation theory: If the Δ takes information he was supposed to keep
secret, he can be convicted.
1. However, the court says this wasn’t presented to the jury, so they can’t
uphold a conviction based on that theory.

d. Dirks v. SEC
i. Δ knows that a company is engaging in fraud which caused their stock price to be
inflated.
1. The company lied about the value of their assets.
2. It’s a huge overstatement because when the fraud is revealed, the stock
price falls by over $11.
ii. Secrest, the insider, tried to go to the SEC and the media, but they won’t listen.
So, Secrest goes to Dirks, a broker, and gets him to disseminate the information.
iii. Dirks now gets prosecuted for insider trading – no good deed goes unpunished.
iv. Secrest, unlike Chiarella, has a duty to disclose because he’s an officer of Equity
Funding, the investment company committing fraud.
1. Therefore, Secrest had a duty to disclose material facts to the buyers and
sellers of the stock.
v. Why does Dirks, the broker, have a duty?
1. He’s a tippee. A tippee who receives information from an insider inherits
his duty to disclose. He’s a quasi-insider.
vi. The SCOTUS is inventing this doctrine because of all the potential for abuse
under Chiarella.
vii. Under Dirks, anyone who has a connection to a company and gets material
nonpublic information owes a duty to the company and its shareholders.
viii. Quasi-Insider Test:
1. Insider must breach a duty.
2. Tippee must know of the breach.
3. Insider must gain from the breach.

98
ix. The court says that Dirks is participating in Secrest’s malfeasance, so the tippee
becomes part of the insider trading.
1. Arguably, Secrest breached a duty to the shareholders of Equity Funding
by divulging the information.
2. However, Secrest didn’t gain, so the third prong of tippee liability isn’t
met.
x. If Dirks had given Secrest any kind of benefit in exchange for the information,
Secrest would have gained and Dirks would be liable as a tippee.
xi. Remote tippees (those who find the information on the street or something) can
also be liable, but only if they were aware of the breach.
xii. If Secrest’s wife got a benefit, that would count for the purposes of tippee
liability.
xiii. Is it a good or bad rule that if an insider can’t trade, you can’t trade?
1. Arguably, under that rule, analyst meetings are illegal. The company
isn’t allowed to give analysts material non-public information.
e. Misappropriation Theory: United States v. O’Hagan
i. O’Hagan is a lawyer employed by a law firm representing the bidder, a British
company.
ii. The stock of the target company rose from $39 to $60.
iii. O’Hagan buys stock of the target, then sells on the day of the takeover bid,
making $4 million.
iv. There’s no doubt O’Hagan has breached duties owed to the law firm and their
client, the bidder. The question is whether O’Hagan lied.
1. To be guilty of securities fraud, you have to have lied.
2. One could say that O’Hagan owed a duty to disclose and was silent.
3. The court, however, says he lied because O’Hagan impliedly promised
loyalty without really meaning it.
v. If O’Hagan had told his law firm he intended to defraud the target company and
short their stock, §10b wouldn’t apply because there’s no lie.
vi. Why stretch §10b to catch O’Hagan?
1. To plug the holes in Chiarella.
2. It’s a bad thing that people can legally game the securities markets as
long as he owed no duty to the company whose stock is being sold.
vii. Congress, in 10b, gave anyone involved in the transaction the right to sue an
inside trader. Damages are limited to the ill-gotten gain. The π gets their
proportionate share.
viii. §14e-3 is an SEC rule that makes it illegal to trade based on material nonpublic
information that comes from either the bidder or target without disclosure. This
gets around the duty requirement.
1. It puts the person with information in the shoes of the insiders – they
can’t trade, you can’t trade.
ix. §14e-3 goes beyond penalizing fraud. The SEC is permitted to promulgate rules
that are “reasonably designed to prevent fraud” in connection with a tender offer.
1. Penalizing the use of information even without fraud could be a
reasonable prophylactic against fraud.
x. With this case, the SCOTUS filled in all the loopholes:
1. Quasi-insiders are liable

99
2. Tippees are liable
3. You can be liable under misappropriation theory
4. You can be liable under 14e-3 even without fraud (if it’s in the tender
offer context)
f. Hypos:
i. If you find a letter on the street from a CEO announcing a tender offer, it’s illegal
for you to trade on it per 14e-3.
ii. If you overhear some CEOs in a restaurant talking about a new invention that
will make their stock price soar, it’s OK to trade on that material nonpublic
information because there’s no tender offer involved and you didn’t commit
fraud.
1. In sum, when the material nonpublic information involves a tender offer,
you can’t trade on it in any circumstances.
g. 16(a)
i. Covered companies have to register with the SEC.
ii. Directors, officers, and people with more than 10% of the company’s stock are
“covered individuals” who must disclose all their trading.
h. 16(b)
i. A covered individual can’t buy and sell in any 6-month period and make a profit.
If you do, then the company can sue for that profit.
1. It doesn’t matter if the insider had material nonpublic information, so it’s
overinclusive in that respect.
2. If you make a profit based on trades that are 6 months and one day apart,
you are fine even if you are loaded with material nonpublic information.
3. Due Process requires that an economic statute have a rational basis to its
purpose, so 16(b) doesn’t violate the Constitution.
ii. Judge Hand’s 14e-3 formula calculates profit based on the difference between the
highest sale price and lowest buy price in the 6-month period. Then, the
difference between the second highest sale price and the second lowest buy price.
Losses don’t offset gains that are disgorgable.
1. The whole series doesn’t have to be within 6 months. Just the matched
pairs must be within 6 months of each other.
iii. Why have such a harsh rule? Even if the insider lost money on the trades, his
insider trading may have prevented greater losses.

i. Kern County Land Co. v. Occidental Petroleum Corp.


i. Timeline:
1. 5/8/67: Occidental makes tender offer for Kern
2. 5/19/67: Tennaco announces a merger with Kern
3. 6/2/67: Occidental receives option
4. 7/17/67: Merger approved
5. 8/30/67: Transaction closes
6. 12/11/67: Tennaco exercises options.
ii. Δ’s stock purchases:
1. 4/7: 0.04%
2. 5/8: 11.7%
3. 5/31: 6.11%

100
4. 6/17: 0.3%
5. Total: 20.57%
iii. The transaction that puts you over the 10% threshold doesn’t count – you can’t
be liable for it under 16(b). So, they’re liable for everything after 5/8.
iv. Δ got value for their stock when the merger finalized on 8/30/1967.
v. Issue: Does this count as a sale under 16(b)?
1. Courts says no. They say that the disposal of Kern stock was involuntary,
so it doesn’t count as a sale (even though there’s no voluntariness
requirement in the statute).
a. Also, the Δ had no access to inside information.
2. Because the transaction wasn’t voluntary and Δs lacked insider
information, the Court decides 16(b) doesn’t apply.
vi. The Court also says the option purchase doesn’t count.
1. Court says the option doesn’t count until it’s exercised.
a. The option was exercised in December, so ti fell outside of the 6-
month window.
2. Also, Occidental wasn’t a 10% owner of Tennaco, so that also shouldn’t
fall under 16(b).
vii. If you were a director or officer when you bought, it’s still a 16(b) violation if
you resign and sell within 6 months.
1. The opposite isn’t true. If you bought stock, became a director, then sold,
then there’s no 16(b) violation. This is because the insider information
wasn’t there for the purchase.
2. Buy  Resign  Sell = Insider trading
3. Buy  Become director  Sell = Not insider trading
4. 10% owner  Buy  Sell = Insider trading
5. Buy  10% owner  Sell = Not insider trading.

101
VII. Contests for Corporate Control
Z. Proxy fights
a. A proxy fight is a battle to get enough shareholder votes to remove the directors of a
corporation. This method of wresting control of a corporation fell out of favor because of
management’s ability to control the proxy machinery (when shareholder meetings take
place, etc.) and because shareholders tend to be pro-management.
b. In its stead, the hostile takeover (in the form of a tender offer to shareholders) became the
preferred method.
c. However, in recent years, employee-stockholders and institutional stockholders have
become more aggressive about challenging directors’ authority. Also, many states have
given management many tools and defensive mechanisms against a hostile takeover
(white knights, poison pills, etc.).

AA. Tender Offers


i. A tender offer is an offer by a bidder to purchase a corporation’s shares directly from its
shareholders.
ii. Defensive tactics against tender offers used to be challenged as market manipulations under
federal securities law, but Schreiber v. Burlington Northern put a stop to that. Instead, such
challenges must be under state law.

iii. Unocal Corp. v. Mesa Petroleum Co.


1. Once you buy more than 5% of a company, §13(b) requires all sorts of reporting
regarding your plans for the company, etc.
2. When making a tender offer, 16(b) requires additional reporting.
3. Mesa paid $35 per share to buy 13%, then he made the tender offer.
4. There are many reasons why Mesa wants to buy as much stock as he can in the 10-day
window before the reporting requirements kick in.
5. The tender offer plan:
a. 13% (bought in the 10-day window) + 37% (cash tender offer, stage one) +
50% ($54 in junk bonds tender offer, stage two)
b. (Junk bonds aren’t necessarily bad investments – they pay more because
they’re riskier)
i. Mesa’s valuation of the bonds is an opinion rather than a fact. The
interest rate must compensate for the risk.
c. To force the minority shareholders to sell out for bonds, Mesa’s plan is to
merge the companies.
i. To approve the merger, the merged company’s board must approve,
then a majority of outstanding shares entitled to vote must approve.
d. Mesa borrowed $3 million to take over Unocal, and the loan is conditioned on
his owning 100% of Unocal.
e. An appraisal wouldn’t help the minority sharehodlers because it would give
fair value for the shares prior to the tender offer, which would be $35.
6. The board of Unocal says the tender offer is too low despite the 53% premium.

102
7. To thwart the tender offer, Unocal board began a self-tender, where the corporation
would try to buy back shares.
a. It’s a discriminatory self-tender – Mesa was excluded from the offer because
he owned too many shares.
8. Holding 1: The court decides the discrimination amongst shareholders is OK because
the court will look at the impact on the shares, not on the shareholders (see also
Sinclair Oil).
a. They’re not discriminatory against the shares, just against Mesa because he
owns too many of them.
9. The self-tender is for 49.9% of the shares at $72 bonds per share, but only if the first
stage of the tender offer goes through.
10. Prior to this case, the Unocal board’s self-tender would be judged by the business
judgment rule.
a. They are trying to preserve their control. Is that self-dealing?
i. No: Their positions aren’t that valuable to them on a monetary basis.
Their shares are worth the same as everyone else’s (see Sinclair Oil
again).
ii. Yes: They’re helping the CEO, whose job is very valuable. They’re all
scratching each other’s backs.
11. If there is self-dealing, then the entire fairness test would apply (fair price and fair
dealing).
a. This would be very difficult for Unocal because $54 was a very fair price.
12. Holding 2: The court doesn’t want to apply entire fairness and it can’t apply BJR, so
they invent a new test:
a. (1) The directors had reasonable grounds for believing that a danger to
corporate policy and effectiveness existed because of another person’s stock
ownership.
i. Satisfied by a showing of good faith and reasonable investigation.
b. (2) The defensive measure is reasonable in relation to the threat posed.
13. Applying the test…
a. Threat 1
i. Unocal said the price offered was inadequate, 53% premium
notwithstanding, and the court agreed based on questionable data from
investment bankers, who said the liquidation price would be higher
(even though the board had no intention of liquidating).
1. If all it takes is an investment banker’s opinion to show you’ve
acted reasonably, then this test is no different from the BJR.
(Per Van Gorkum, BJR comes with a duty of care anyway).
ii. Accepting the I-banker’s opinion as true, the defensive measure must
be reasonable – they’re offering a price in their self-tender that’s
consistent with thte I-banker’s opinion.
b. Threat 2
i. Unocal also said the tender offer was a threat because it was coercive.
1. The bonds weren’t a good consideration for the shares,
according to Unocal.
2. They say the bonds weren’t really worth $54.
a. Unocal has no proof of this.

103
b. Yet, the court allows this argument and says the two-
step offer was coercive.
ii. Here, the defensive mechanism is not reasonable for the threat. There’s
no reason to pay $72 to compensate for bonds that are supposedly
worth $54.

c. Threat 3
i. Greenmail. The purported bidder must be bought out by the target with
money not shared with other shareholders.
1. DE law says that Greenmail is legal as long as the board is
saving shareholders from something worse.
ii. How is greenmail a threat, though, when it’s entirely optional? To
escape greenmail, just don’t pay greenmail, ya turkey!
iv. Revlon v. Macandrews & Forbes Holdings
1. Pantry Pride wants to take over Revlon using a “bear hug.” The first approach
management and, if they don’t agree, they threaten a hostile takeover.
a. Original offer to the board: $47.50
2. Defensive measures:
a. Back End Rights Plan: An offer from Revlon management to exchange shares
for a note ($65 note).
i. The back end rights plan is triggered by someone purchasing 20% of
the company. The person who buys the 20% isn’t eligible for the back
end rights plan.
1. The purpose of this is to price out Pantry Pride. However, this
prices the company at approximately $56 per share. It doesn’t
stop Pantry Pride because they believe the company is worth
about that much.
2. The real purpose is to make the shareholders think twice about
accepting a tender offer less than that.
b. Exchange offer: 10 million shares at $47.50 note, plus $10 preferred shares =
$57.50
i. The purpose of this is the covenant in the note which prohibits Revlon
from taking on more debt. Because Pantry Pride needs to borrow
money to complete its takeover.
3. It wasn’t necessary for the result of the case, but the DE supreme court ruled on the
validity of the defensive measures anyway.
4. Is the back end rights/exchange offer plan legal under Unocal?
a. Revlon says the threat they’re responding to is 1) inadequacy of price and 2)
the threat of the “bust-up” takeover.
5. Why is the company being run at $55 per share when it’s worth more busted up?
a. Executives of bigger companies get paid more.
6. Unocal Prong One: Due diligence as to the threat.
a. The I-bankers’ opinion regarding the liquidation value is sufficient to justify a
perception of an inadequacy threat (see also Unocal).

104
i. This is not going to be scrutinized by the court – just like in Van
Gorkum, where the I-banker’s opinion alone is enough to satisfy the
duty of care.
b. Is it OK to consider the interests of non-shareholder constituencies when
defending against a takeover?
i. Unocal says yes, other constituencies’ interests can be taken into
account in the determination of a threat. Therefore, the threat of the
bust-up is adequate for this prong.
7. Unocal Prong Two: Defensive measures proportional?
a. The court seems to say that stopping an offer altogether (by preventing Pantry
Pride from borrowing money in this case) is proportional.
b. In response to this defense, PP lowered its offer. But then they unilaterally
raised their offer to $53.
c. While the above offer was pending, the board found a white knight to merge
with (Forstmann, who offered $56).
i. Why find a white knight? To appease shareholders, who have been
teased with enticing offers.
d. PP, in turn, raised its bid to $56.25 and said they’d offer $0.25 more than any
bid Forstmann makes.
e. To get Forstmann to make the $57.25 offer, Revlon promised: 1) A no shop
clause, 2) a lock-up option, and 3) bust-up fee of $25 million if the deal doesn’t
go through.
i. The no shop clause promises Revlon won’t give information to any
other bidders.
ii. The lock up option promised to sell Revlon’s best assets to Forstmann
cheaply.
iii. If PP won, they’d have to pay for those commitments to Forstmann.
f. PP then offered $58 conditioned on Revlon canceling the covenants and the
options for Forstmann.
g. It’s not illegal per se to have an uneven playing field for bidders – all they have
to show is that it was to maximize shareholder value.
i. Because the favoritism resulted in a higher bid, couldn’t the board
argue that their tactics maximized shareholder value?
1. The question is whether Forstmann would have raised its bid
even without the favoritism.
h. The point of giving favoritism is to gin up bidders to raise the price.
i. Once the bidding is already started, the court said the favoritism was
unnecessary to facilitate bidding.
8. Holding: The court imposes a stricter standard than Unocal. The board must show that
the tactics were designed to increase SH value – this is even tougher than the entire
fairness test.
9. Revlon also argued they were looking out for non-SH constituencies in resisting the
takeover.
a. However, at the point that the company was legitimately for sale, shareholders
can no longer benefit from helping non-SH constituencies. Therefore, you can
no longer do things in the interest of non-SH constituencies.

105
i. This probably means you can never benefit non-SHs unless it has a
connection to SH benefit.
b. It’s an open question whether this part of Revlon overrules Unocal’s holding
that non-SH constituencies can factor into a board’s perception of a takeover
threat.
c. Rule: What’s clear is that once the company is for sale, you can’t take non-SH
constituencies into account at all.

106

You might also like